× Limited Time Offer ! FLAT 20-40% off - Grab Deal Before It’s Gone. Order Now
Connect With Us
Order Now

MGT604, MGMT6011 Strategic Management Assignment Sample

Stakeholder Meeting Simulation and White Paper

PART A: Preliminary Outline (Hurdle)
 

Question:

Pro-Go Pty Ltd Scenario

The scenario provides you with a brief overview of a hypothetical project for a real-life organization.
Be aware that the scenario may not cover every detail that you will need to address in the White Paper, in which case, you will need to conduct additional research, including further research into the industry concerned. [Pro-Go Pty Ltd is a privately-owned manufacturer of small, high-quality action cameras based in Geelong, Victoria. Founded in 2004 it has enjoyed continuous year-on-year growth until the last financial year. Pro-Go has a track record of innovation, and technology that provides excellent quality results in a small package able to operate with small batteries for extended periods of time. The video files these cameras produce are known for having superior colour, and operate effectively in lower lighting conditions than competitor’s cameras. They are also marketed on the basis of their extreme ruggedness, and reliability. Much of the technology used in its devices has been patented by Pro-Go and is not currently available to other manufacturers.

The Pro-Go management team has recognised that the ability to capture videos is now a feature of most phones. It suggests to them that the future of the stand-alone camera may be limited. Pro-Go’s CEO has requested the executive team meet independently to develop a strategy to guide the company over the next five years.]

Deliverables

The stakeholder meeting will discuss and agree on three strategy options for Pro-Go. In doing so, the Stakeholder meeting may consider strategy options that exist in adjacent markets or with related but different products or technologies. Remember to apply strategy concepts in your stakeholder meeting discussions to identify strategy options.

The deliverables required of each student are to produce a white paper covering the following:

1. Nominate the preferred strategy option and apply a strategy framework(s) to analyse and justify why you believe it is the best strategy option; and

2. Discuss the implications specifically relating to your part of the business, and what challenges these might represent. Separate the discussion into short-term and long-term issues.

Students are to address the assessment task from the perspective of their own functional area of responsibility. Integrate theory in your white paper.

Answer:

Introduction

The company, Pro-Go Pty Ltd. was started in 2004. The company is a manufacturer of high-quality cameras. This company has seen growth since its beginning but recently has seen a decline in its growth and market share. This short report will suggest some strategic plans for the company among which the best option is chosen.

Report Aim

The aim of the report is to create a sustainable strategy for the company for 5-year period.

Discussion

As a general manager of the company, following strategies has been suggested for Pro-Go Pty Ltd.:

1st Strategy: To have a collaboration with smartphone companies like Motorola, Huawei, HTC and Nokia to install their Pro-Go Pty Ltd.’s cameras in the smartphones. This can induce the promotions and sales of both smartphone companies and the camera company (Todorov& Akbar, 2019).

2nd Strategy: Tie ups with the companies to create new products like smartphones, smartwatches, andsmart headphones to gain new market share (Markides, 2016).

3rd Strategy: Rebranding can be done with the help of influencer marketing on Instagram. This is a very effective strategy where influencers can be hired, and they can promote the products of Pro-Go Pty Ltd. With the special code given to the customers if they purchase the products by using that code (Mintzer& Entrepreneur Media, Inc., 2018).

Strategy option : Preferred one.

Three strategies are suggested for Pro-Go Pty Ltd. Porter generic strategy framework has been used here to get the preferred strategy option.

Source: Business Balls

 

This model shows the way in which the company should move forward so that it can an edge over its competitors (Firoz Suleman, Rashidirad & Firoz Suleman, 2019). It also helps in understanding how a company can increase its market share or sales in the market. Pro-Go Pty Ltd. develop high quality action cameras which was its USP since the start of the company but due to the decline in its sales, the company should go for tie ups with other companies which producessmartphones, smartwatches, and headphones (Xhavit, Naim & Marija, 2020). This is because the high-quality action cameras are now not in demand. The industry has become saturated now. Differentiation strategy would be the best strategy as it would broaden up the scope of the company and it will enhance the competitive edge in the market which is highly essential for the company right now (People Matters, 2021).

The company could allure broad market, millennials and also adults who are fitness freak, mobile phone users and music lovers. This would broaden up the scope of the company.

Short Run Implication of Strategy

The company can successfully implement such strategy. However, bringing companies and having collaboration with them could really take much time. It would be time taking to be in collaboration with the companies and to set up a deal fixing all the terms and conditions on the collaboration (Bolland, 2017).

Long Run implication

I as a general manager of the company will assure that the strategy remains sustainable. For this, the company would have to provide training sessions to its workers for dealing with new technology. The company would have to successfully implement change management strategies so that the strategy remains sustainable.
 

References

Bolland, E. J. (2017). Comprehensive strategic management : a guide for students, insight for managers. Emerald Publishing Limited.

Firoz Suleman, M., Rashidirad, M., & Firoz Suleman, S. (2019). The applicability of porter's generic strategies in pure online firms: a case study approach. Strategic Change, 28(3), 167–176. https://doi.org/10.1002/jsc.2258

Markides, C. C. 2016. Diversification Vs Focused Business Strategy. Retrieved from https://www.entrepreneur.com/article/277302.

Mintzer, R., & Entrepreneur Media, Inc. (2018). Entrepreneur voices on strategic management (Ser. Entrepreneur voices). Entrepreneur Media.

People Matters. (2021). Fujifilm's perfect shot at success - Diversification of business. Retrieved from https://www.peoplematters.in/article/c-suite/fujifilms-perfect-shot-success-diversification-business-13258?media_type=article&subcat=leadership&title=fujifilms-perfect-shot-success-diversification-business&id=13258

Todorov, K., & Akbar, Y. H. (Eds.). (2019). Strategic management in emerging markets : aligning business and corporate strategy (First). Emerald Publishing Limited

Xhavit, I., Naim, M., & Marija, T. L. (2020). Linking porter’s generic strategies to firm performance. Future Business Journal, 6(1), 1–15. https://doi.org/10.1186/s43093-020-0009-1


 

Read More

MGT601 Dynamic Leadership Assignment Sample

Assessment Task:

Your task in Assessment 1, Part A, is to complete the Gallup Strengths Finder and submit a brief report on the results and their implications for your development as a leader.

Analyzing Results:

When you receive your results, look at your report and read the background document on the Gallup psychometric assessment. You should then prepare a 500-word report which covers the following issues:

1) A brief discussion of the theoretical foundation of the Gallup Strengths Finder (Positive Psychology) and why it is important

2) A brief description of how your results reflect your approach to personal power. Provide a copy of your report as an Appendix. It will not be included in the word count

3) Your initial reactions to your results: where have you seen these strengths being used in your life to date?

4) Your initial thoughts on the fit between your authentic self (as represented by your strengths) and the way you have approached the university degree for which you are currently studying.

Please note that it is quite difficult to cover these four points in 500 words – pre-plan what you intend to say and revise your work repeatedly to increase the impact of your writing. Avoid unnecessary discussion and ensure every sentence contributes something positive to the discussion, and is aligned with the requirements of the assessment brief.
 

Answer:

Theoretical Framework of the Gallup Strengths Finder

The Gallup Strengths Finder is a renowned online platform to recognise multi-dimensional skills, abilities, and strengths of an individual based on positive psychology. Positive psychology enables a person to discover talents and strengths that include courage, hope, resilience, confidence, spirituality, and so on (Guay Fre?de?ric, 2017). The Gallup Strengths Finder worked on areas like the development of talents and strengths, engagement of the employees and customers.

Description of the Results on Personal Approaches

This online instrument's assessment helps an individual identify five significant strengths and talents through strength-based development (Salicru, 2017). After using Gallup Strength Finder, I have recognised my five strengths as Ideation, Futuristic, Harmony, Input, and Individualisation. I feel delighted when I discover the complexity of a simple concept that comments me to identify new ideas and thoughts regarding the idea. My passion for ideas will help me to see things from a different angle. Ideas can enable a person to conceptualise a matter from different perspectives (Aneesh, Tony, & Sudhesh, 2018). So, ideas make me creative and conceptual. The futuristic aspect of mine makes me peer over the present scenario of incidents. The outcome shows that the futuristic element of mine allows me to see things in a detailed manner that pulls me to visualise things from a futuristic point of view. Again, in the context of harmony, I prefer common grounds when people around me are holding different opinions and concepts that might lead to conflict. My preference towards harmony will help me to clarify the conflicted grounds of people and thus reach a common goal and objective. Various inputs and information can help a person to remain updated with different topics and areas (Danukarjanto, 2017). As I am a very inquisitive person, I prefer to gather information about different topics to remain updated, which will enable me to someone to give information about any subject. The strengths of individualisation will allow me to look at the persons by finding out their individual qualities and differences. This quality of mine helps me to build efficient and productive teams.

Reactions on the Results

As I get the results from Gallup Strengths Finders, I became very much happy with the outcomes. I had no idea about my strengths and abilities before the assessment from this online instrument. But after I found out that I have some strategic strengths and capabilities, it has formed self-confidence that would help me in my personal and professional life in the future. These identified strengths, I have witnessed when I engaged in handling community event at my locality with some neighbour colleagues. Therefore, I helped in developing ideas and engaging volunteers to successfully conduct the event.

Initial Thoughts

The first initial thought after getting the results was that as I am a student how can I apply those strengths in my student life and projects that I got from the university. So, I found out that I could use those strengths to increase my productivity and creativity to reduce the study pressure from my authentic self. It also gave me chances to get various information about different matters that will help me to achieve my goals through the university degree.  
References

Aneesh, K. P., Tony, S. G., & Sudhesh, N. T., (Eds.). (2018). Character strength development: perspectives from positive psychology. SAGE. Retrieved on: 17 June 2021, retrieved from: https://lesa.on.worldcat.org/oclc/1053998936

Danukarjanto, L. (2017). Stuff I wish I knew earlier: unlock your career potential. Iguana Books. Retrieved on: 17 June 2021, retrieved from: https://lesa.on.worldcat.org/oclc/1009270606

Guay Fre?de?ric (Ed.). (2017). Self, driving positive psychology and well-being (Ser. International advances in self-research, volume 6). Information Age Publishing. Retrieved on: 17 June 2021, retrieved from: https://lesa.on.worldcat.org/oclc/1002693073

Salicru, S. (2017). Leadership results: how to create adaptive leaders and high-performing organisations for an uncertain world. John Wiley & Sons, Incorporated. Retrieved on: 17 June 2021, retrieved from: https://lesa.on.worldcat.org/oclc/1002695818

 

Read More

MGT605 Business Capstone Project Assignment Sample

Question:

Subject Learning Outcomes

SLO A. Develop and articulate a theoretical or practical perspective on management issues through a substantial applied project.

SLO B. Demonstrate collaborative leadership and ownership in the preparation of a significant original body of practical or theoretical work.

SLO C. Use specialist research skills to analyse a complex management problem and then synthesise the research, communicating it effectively to both specialist and lay audiences.

Instructions:

In this assessment, you are required to propose how you are going to study your capstone project. Before you can propose, you first need to identify a project idea on which to develop your project proposal. Your project proposal must include the opportunity statement or pain/problem statement.

The capstone project proposal serves two basic functions. Firstly, it presents and explains how you are going to study and analyse the issue of your capstone project. This is essential to make clear what you aim to achieve by the end of this study period, what data you will collect and analyse and what MBA subject theory applies to your project.

Secondly, how you will conduct the capstone project. This is an account of the steps you will take until you produce the final report. This includes a timeline. The timeline will help you track your progress.

With this information, your facilitator will be able to guide you concerning the suitability and feasibility of your capstone project idea.

Answer:

Introduction:

Due to the outbreak of COVID-19 pandemic, many countries adopt the lockdown regulation to maintain physical distance with each other. The drastic outbreak of this deadly virus has declined economic growth of counties by hampering business and social activities. Australia also imposes lockdown and consequently companies are forced to shut down or reduce production temporarily during this time. Tech industry is Australia has also significantly experienced the negative impact of COVID-19 pandemic. The pandemic leads tech companies to adopt remote working strategy (Wang et al., 2021). Accenture has more 4500 employees in Australia. Many Australians also work at client sites with the organization. During pandemic, the company considers remote working as the necessity instead of a choice. However, managing isolation and disengaged of employees through virtual team collaboration also becomes an important issue for managers during the current time.

Background:

It is known that COVID-19 opens a new dimension of work culture, which is remote working. The working nature of Accenture allows remote working and hence more than 500000 employees work remote working across the world (Accenture, 2021). The global IT organisation has adopted a digital work program to communicate with remote employees to increase their capability of being efficient and productive. During the unpleasant situation of pandemic, the company provides significant effort to keep its work force productive across the world and this is also true for employees in Australia. From the report published by the company on remote working, it is seen that 94% of the company’s global office employees work remotely (Accenture.com, 2021). Moreover, it is also seen that employees are seeking more support from employers as they need greater connection while separation from work also causes an issue for them.

Aim of the Project:

The project proposal is designed to find challenges in managing employees at Accenture during COVID-19 pandemic and to find out effective solution. The project is taken under consideration for knowing whether managing isolation and disengaged of employees by team virtually is a serious issue or not as the remote working can be a new future for organisations (Accenture, 2021).Working from home remotely creates the feeling of loneliness and lack of work motivation among employees. In this situation, managing Isolation and disengaged feelings among workers become a serious issue among employers.Whether managing team virtually is practical or not, depends entirely on the workforce along with the facility of work. However, managing work force is a great challenge as employers cannot practice the existing workplace policies when employees are working from their home. The project addresses the MBA subjects that emphasis on employee motivation, satisfaction and engagement.

Therefore, the main aim of the recent study is to analyse the issue and solve the problem with proper managerial strategies so that employers of other sectors be motivated and engaged their remote employees with work in any situation.

Literature Review:

Theories:

Maslow’s Hierarchy of Needs: According to the motivational theory, developed by Abraham H Maslow, people have a hierarchy of needs. According to this theory, actions of human are influenced to achieve certain needs and when one need is fulfilled, the person goes to the next level of the hierarchy. The hierarchy has 5 steps of needs related to physical, security, social, esteem and self-actualisation (Jonas, 2016). Employees work to meet their basic needs like food, clothing and shelter. At the same time, they need job security along with safe work environment. Moreover, employees need to belong in a team and want to be an important part of it. Employees seek valuable treatment from the organisation and rest of the team members. This is self esteem which leads employees to be successful in their work (?tefan, Popa & Albu, 2020). Lastly, self-actualisation occurs when employees’ all the previous become fulfilled. This stage leads the employee to become more creative and progressive in its career.

Two factors model of job satisfaction: The theory was developed by Herzberg who recognises some factors that can either satisfy or dissatisfy employees. Satisfactory factors are recognition, achievement, and responsibility (Velmurugan & Sankar, 2017). On the contrary, dissatisfactory factors are salary, work condition, company policies, safety, security and supervision. The theory tries to address work motivation from the perspective of work satisfaction. From the finding of Alshmemri, Shahwan-Akland Maude, (2017) it is seen that the factors that cause dissatisfaction among employees are called hygiene that are the part of the aspect in which the job is done. On the other side, motivation are the factors like recognition, achievement, increased responsibility, interesting work, advancement etc which are intrinsic to work.

Findings of others:

From the finding of Toniolo-Barrios and Pitt (2021) it is seen that homes do not allow employees to balance personal and professional life properly and consequently their work productivity have declined. Employees face lack of motivation and increasing work stress during this situation. In another finding, Wang et al., (2021) also observed that employees’ performance depends completely on their well-being and challenges that they experience during work.

Literature Gap:

The studies focus theories related to employee motivating and satisfaction and also focus on challenges that employees and employers face in conducting remote work during pandemic. However, there are lack of studies that focus on the specific issue of motivating isolated and dissatisfied employees of a large tech company, which is, Accenture through virtual communication during pandemic.

Method:

According to the WHS laws, each organisation needs to take care of the health and safety of their employees. This responsibility leads employers to protect employees from possible risks associated with COVID-19 by implementing proper safety measures. To avoid the risk and to continue working, organisations encourage employees to work from their homes (Accenture, 2021). However, the situation also generates the risk of managing, encouraging or directing workforce effectively to work. Perceived social isolation during pandemic has significant impact on humans, especially at psychological grounds. To find out challenges in managing isolation and disengaged feelings among workers at Accenture through virtual team collaboration, the project proposes a research method that includes research design, research approach and data collection as well as analytical technique.

In general, a research method can be either quantitative or qualitative or mixed in nature. In this study, the researcher will apply qualitative research method as quantitative research method only helps the researcher to get objective outcomes only. Qualitative research method focuses on in-depth analysis and helps the researcher to obtain outcome in descriptive way. This method will allow the study to find why virtual team collaboration faces challenges in managing employees who feel isolated and disengaged due to remote working system.

Qualitative research design varies on the basis of the method is applied. Each research design follows a formal, systematic and objective process for collecting data. In this research, the researcher will apply descriptive research design that will help to explain the research aim (Doyle et al., 2020). This research study is considered as the simplest analytical method. Hence, it is useful in shedding light on the selected research issue by collecting and analysing data.

In the study, the researcher will apply secondary method to collect data (Kizil, Muz?r & Y?lmaz, 2021). In this type of research, researcher can consider newspapers, magazines, reliable publications that focus on motivating employees who work remotely. Moreover, the report will consider report published by Accenture related to the recent pandemic on the company performance.

The entire study will follow deductive research approach, which leads research to develop hypotheses depending on existing theories and to design a research strategy for testing these hypotheses (Casula, Rangarajan & Shields, 2021). The approach will help to consider motivational and satisfaction theories of employees in organisations to find challenges faced by Accenture and the way these challenges can be solved. The outcome of the capstone project will be applicable in finding the casual relationship between isolation and disengaged feelings among workers and virtual team collaboration in Accenture. The approach will help the study to find and establish a valid outcome.

The study will collect performance report of Accenture for last 5 years including profit and company’s costs on employees. Moreover, the project will also consider other relevant articles that discuss on relevant topic emphasising on tech companies. The study will conduct content analysis to get the desired outcome.
From the finding, it can be seen that employees will experience isolation and dissatisfaction in their jobs due to lack of recognition through virtual team collaboration. Moreover, lack of job security will also force them to remain dissatisfied with their jobs.

Conclusion:

The present study focuses on managing isolation and disengaged feelings among workers at Accenture through virtual team collaboration. The study will focus on the employee motivation, engagement and satisfaction topic of MBA. The study finds that the outbreak of COVID-19 pandemic has forced all employees to work from their home. In this situation, managing employees by keeping them motivated and satisfied with works become a serious issue for employers in Accenture. Hence, through the study, the researcher will focus on the issue. Here, the study considers secondary qualitative research method that will be analysed by considering descriptive research design and deductive research approach.

Indictive Framework:

Task

Start week

End Week

Duration

 

Topic Selection

1st

1st

2

 

Finding research aim

1st

1st

1

 

Finding related theories and literature

1st

2nd

4

 

Analysing process of data analysis method

2nd

2nd

2

 

Submitting project proposal

3rd

3rd

1

 

Analysis of more literature

4th

7th

11

 

Data collection and analysis

8th

12th

15

 

Discussion and findings

13th

15th

9

 

Conclusion and recommendations

16th

20th

25

 
         

 

References:

Accenture. (2021). COVID-19: Remote Work Solutions | Microsoft Teams Case Study | Accenture. Retrieved from https://www.accenture.com/in-en/case-studies/energy/national-oil-company-remote-workplace

Accenture. (2021). Enabling Effective Remote Working | Accenture. Retrievedfrom https://www.accenture.com/us-en/blogs/how-accenture-does-it/how-accenture-enables-effective-remote-working

Accenture. (2021). Ready and Remote | Global IT | Accenture. Retrieved from https://www.accenture.com/cr-en/case-studies/about/ready-remote

Accenture.com. (2021). Retrieved from https://www.accenture.com/_acnmedia/PDF-152/Accenture-Final-Remote-Working-POV-April-2021.pdf.

Alshmemri, M., Shahwan-Akl, L., & Maude, P. (2017). Herzberg’s two-factor theory. Life Science Journal14(5), 12-16.

Casula, M., Rangarajan, N., & Shields, P. (2021). The potential of working hypotheses for deductive exploratory research. Quality & Quantity55(5), 1703-1725.

Doyle, L., McCabe, C., Keogh, B., Brady, A., & McCann, M. (2020). An overview of the qualitative descriptive design within nursing research. Journal of Research in Nursing25(5), 443-455.

Jonas, J. (2016). Making practical use of Maslow’s Hierarchy of Needs theory to motivate employees: a case of Masvingo Polytechnic. Journal of Management & Administration2016(2), 105-117.

Kizil, C., Muz?r, E., &Y?lmaz, V. (2021). Auditing Techniques to Minimize Accounting Related Fraud and Errors: A Qualitative Analysis with the Interview Method. Emerging Markets Journal (EMAJ), University of Pittsburgh Press (USA)11(1), 95-103.

?tefan, S. C., Popa, ?. C., &Albu, C. F. (2020). Implications of Maslow’s hierarchy of needs theory on healthcare employees’ performance. Transylvanian Review of Administrative Sciences16(59), 124-143.

Toniolo-Barrios, M., & Pitt, L. (2021). Mindfulness and the challenges of working from home in times of crisis. Business horizons64(2), 189-197.

Velmurugan, T. A., & Sankar, J. G. (2017). A comparative study on motivation theory with Maslow’s hierarchy theory and two factor theory in organization. Indo-Iranian Journal of Scientific Research1(1), 204-208.

Wang, B., Liu, Y., Qian, J., & Parker, S. K. (2021). Achieving effective remote working during the COVID?19 pandemic: A work design perspective. Applied psychology70(1), 16-59.

 

Wang, B., Liu, Y., Qian, J., & Parker, S. K. (2021). Achieving effective remote working during the COVID?19 pandemic: A work design perspective. Applied psychology70(1), 16-59.

 

 

Read More

MKT600/MKTG6002 Marketing Assessment 1 - Situation Analysis Sample

Question:

Instructions:

This assessment task requires you to develop a Marketing Environmental Analysis that comprises both the micro-environment and macro-environment. The micro-environment analysis involves scrutiny of the internal environment that facilitates specifying the strengths and weaknesses, while, the macro-environment analysis requires the scrutiny of demographic, economic, natural, political, technological, and cultural (DENPTC) dimensions which will assist in specifying the opportunities and threats. Changes within the macro-environmental forces are outside of an organization’s direct control requiring an organization’s marketing strategies to be adjusted to capture emerging opportunities and minimize any potential threats.

Therefore, in this assessment task you will need to:

• Articulate your learning about marketing concepts and principles covered from Module 1 to 3.1 to demonstrate your level of understanding to evaluate the organisation's overall situation;

• Critically analyse the link between marketing theory and practice;

• Demonstrate research skills to reveal the insights;

• Apply appropriate business report writing skills;

• Discuss appropriate strategies with regards to your SWOT analysis.

• Use the appropriate APA style both in text citation and the reference list at the end. You should use at least 10 relevant academic references from leading marketing journals and text books to showcase quality of your research.

Tasks: 1. Discuss the value propositions, core brand values and buyer behaviour of your chosen organisation;

2. Analyse the micro-environment (the company, suppliers, intermediaries, relevant publics, customers and competitors); 3. Analyse the macro environment, that is, demographic, economic, natural, political, technological and cultural (DENPTC) forces to demonstrate their impacts on your chosen organisation's strategies/capabilities to serve the target market (s; 4. List the strengths, weaknesses, opportunities and threats (SWOT) as well as suggest appropriate strategies by incorporating the TOWS matrix to draw from strengths to capture opportunities, to overcome weaknesses and to defend from threats.

Answer:

Introduction

Situation analysis is a primary step while making marketing plan. In this technique, macro and micro environment analysis of market is done to extract information to support marketing strategy. The chosen organization for this report is Tony Ferguson. The company is well known for selling protein shakes to lose weight. The quick early growth of the Tony Ferguson’s weight loss product is the indication of successful overseas expansion plan of the company.

Value propositions, core brand values and buyer behavior

Value proposition is strategic analysis of a company which acts as unique selling proposition for a company. Core brand values and buyer behavior are associated with identification of main services provided by Tony Ferguson, and matching of these services with customer preferences to analyze market gap of that service (Osterwalder et al., 2014).
 

Value propositions

 

Pain

• Customers are facing challenges in identifying appropriate sources of protein which is also healthy in their daily life routine.

• Protein takes time to consume in the body; there is no proper diet available in the market which is a quick source of protein.

• All the protein containing products are not sufficient to fulfill the actual requirement of the protein by the body. The available sources have slow results and not suitable to every person (Murray, 2013).
Gain

• Protein shake provided by Tony Ferguson will solve the problem of carrying it from one place to another. The protein powder is easy to fit in hand bag and requires some filtered water to shake it.

• The availability of this product at every retail outlets and big malls makes it convenient for customers to choose it as a supplementary source of protein.

• The powder can even be packed in small pouches to carry into the pockets. This makes it convenient for people to consume it even in gym or while travelling for business tour where carrying big luggage is not possible (Canster blue, 2021).

Value Proposition

• The Tony Ferguson’s protein shake product has high availability due to its sound distribution channels.

• The different packages of protein shake product makes it convenient for consumers to carry it easily from one place to another and use this product anytime and anywhere.

• After consuming protein shake, consumer doesn’t feel hunger and feel energetic. Another advantage of easy digestion makes this product attractive to those people who don’t have enough time to burn their calories (Murray, 2016).

• Protein gives immense energy to its users, thus consuming it in the morning makes consumers energetic and active (Tony Ferguson, 2021).

Core brand values

Its products can be included in the dining table to improve body alignment, and strength without adding too much weight. Many people can make a profit by adding Tony Ferguson to their menu:

Strong and dynamic adults: Adults who take part in regular exercise may find that Tony Ferguson helps improve body structure by helping to increase body mass, a level at which the body builds appropriate muscles. Tony Ferguson's 20 gram supplement increases protein synthesis when burned before or after the block (Winget, 2017).

Those trying to manage their weight: High-protein diets have been shown to help people feel fuller, which can reduce their weight gain (Kostelijk, 2017). More protein, further calorie reduction has been shown to improve the nature of weight reduction by increasing the deficiency of the muscle-to-fat ratio and also reducing the deficiency of lean muscle mass (Kostelijk and Alsem, 2020).

Mature population: Starting at the age of 40, many adults experience age-related loss of mass, capacity and strength, a condition known as sarcopenia. Exercise and protein supplementation, for example, including Tony Ferguson's protein with every meal, can be an easy way to harden your dick or measure this bad luck (Nguyen, Melewar and Brown, 2019).
 

Buyer Behavior

Protein has seen significant development in recent times, characterized by an uncommon expansion in the apparent lack of development by buyers, awareness and development of plant-based protein use and the subsequent proliferation of high protein and advanced protein elements through various food and drink courses (Miller, Washington and Richard and Associates, 2017). In any case, the development of the previous year is down. Whatever the improvement over the previous five years, general information from consumers about the medical benefits of protein, especially muscle support / building in the past, is much needed, and potential for protein food / drink manufacturers. Also, there are changes between classes, with some logos and even reductions (Miller, Washington and Richard and Associates, 2019).

Interest in protein has skyrocketed in recent years. For example, in 2014, the majority of consumers were shown to be looking for high-protein foods, compared to 39% in 2006, and were shown to be close to 3/4 of consumers using high-protein food types / drinks in 2020. Of these, a quarter indicated that they would expand their use over the same period (Statista, 2020).

Clients see Tony Ferguson as a boon for muscle development and weight reduction. Buyer thinks this option is the best protein boost. Customers love to buy Tony Ferguson’s protein shake because it’s powerful and simple to process (East et al., 2017).

The continuous increase in the demand of protein supplements has been noticed for the past eight years. Like for instance, in the year 2014, more than 50% consumers prefers protein rich foods and supplements, while this figure was 39% in 2006. In 2019, it increases to 75% consumers who take high proportion of proteins compared to other nutrition's in their diet (Schiffman and Wisenblit, 2019).

Consumers perceive that Tony Ferguson is good for muscle growth and weight loss. Buyer considers this product as best supplement alternative for protein. Buyers prefer to buy Tony Ferguson’s protein shake as it is effective and easy to digest (East et al., 2017).

Micro environment

Micro environment analysis has main focus on company itself. Micro environment covers all those external factors which affect the performance of the company on small scale level. The micro environment analysis of Tony Ferguson has been done below:

Company

Tony Ferguson's classic food substitutes are made for lasting slimming potential. It's perfect for those who need to shed pounds but are happy to do it at a steady pace that requires regular festival evaluation every day. The Classic program includes 2 classic snacks, a regular dinner and 2 snacks per day that add up to approximately 1600 calories. Not suitable for pregnant or lactating women and under 18 years of age (Marketing.net, 2020).

Suppliers

Tony Ferguson's suppliers include PSD network ltd., BHI nutrineo GMBH, Foodcom S.A. and many others. Tony Ferguson's suppliers are well organized around the planet. Lion's share of the company's products is traded with various countries (Tony Ferguson, 2021).

Intermediaries

The intermediaries of Tony Ferguson includes malls, retail stores, online platform and sometimes gym instructors also act as intermediaries for protein supplement product.

Customers and competitors

The demand in protein products of the youth population usually extends to competitors as they offer a number of benefits such as lowering cholesterol, working muscles, extending strength, fighting disease, improves invulnerability and reduces pulse. As a result, this basic factor has a positive impact on the protein supplement market figure (Farroq, 2017).

Due to the presence of strong players in the market as well as the presence of several small and medium parts in the market, the global market for whey protein fasteners is very poor. The market is expected to develop at a CAGR of over 10% over the specified period. Manufacturers are trying to coordinate with the growing interest with changes in consumer sentiment, both in terms of taste and location. It can be removed from the pen for foods that contain whey protein, food and beverage manufacturers, supplements, and others generally use whey protein in a variety of details. The main customers of wheeled protein setters are manufacturers of sports support products and manufacturers of supplement products (Business Jargons, 2021).

Macro environment

Macro environment is kind of industry analysis where similar nature of small and large firms taken into consideration. Macro environment analysis is a brief about trend prevailing in the market of particular industry. DENPTC forces have been used to analyze the macro environment of Tony Ferguson.

Demographic

Demographic factor includes population, and size of particular market segments. The main users of protein shakes are athletes and young bodybuilders. Thus increase in their population can also increase the demand for the product (Claessens, 2015).

Economic

A healthy protein shakes have high cost, which sometimes is not affordable for each economic group of the nation. To increase the demand of the product, it is necessary to increase income of lower economic groups.

Natural

Health conscious people prefer natural ingredients in protein shake rather than artificial flavored chemicals. Thus, including more natural ingredients in the product could raise the product demand in the market.

Political

Government policies also play an important role in increasing and decreasing demand of any product. Thus, allowing subsidies on health products could increase the demand of the product at reducing prices.

Technological

Improvement in technology can help company in reducing its overall cost and increase the demand of the product at reducing price.

Cultural

A health conscious and physical fit culture in the society can also increase the demand for protein shakes product in the market (Claessens, 2015).

SWOT/TOWS analysis

It is a form of internal analysis of a particular company. Tony Ferguson’s SWOT/TOWS analyses are focusing on how company’s strength and opportunities can be channelized to mitigate its weakness and threats (Kiron, 2019).

INTERNAL FACTORS

 

 

 

 

 

 

EXTERNAL

 

FACTORS

 

Strengths

·      Produce a quality product that solves the customer's problems

·      Employee reliability is also a strong strength of the organization

·      Attract more consumers to expand their business more than competitors (Mariani, 2017).

Weaknesses

·         Inadequate number of sellers promoting the item.

·         Lack of cash flow to support advertisement.

·         Cases with messengers that accept a new logo.

Opportunities

·         Offers common marketable items.

·         Increase your awareness of leading a better lifestyle.

·         More funding.

 

·      Providing natural products in the market to fulfill customer needs.

·      Rising in healthy lifestyle awareness through employee’s loyalty.

·      Increased sponsorship through gaining more market share (Muth and Zive, 2020).

·         Competition in market while offering natural products

·         Increased in sponsorship can fulfill the capital requirement for advertisement.

·         Increasing awareness will help company in increasing brand awareness.

Threats

·         Competition from previous industry segments.

·         The impacts of the crisis in a crisis that will make individuals spend less.

·         Companies that offer other items for medical benefits

·         Lack of ability to offer borders to customers (Ginter, Duncan and Swayne, 2018).

·      Selling quality products can minimize the competition and avoid adopting discount strategy to attract customers for Tony Ferguson.

·      The worldwide operations of the company can reduce the threat of competition in health benefits product (Academic Pr., 2017).

 

·         Less availability of retailers for product promotion can increase the competition in preexisting markets.

·         In the absence of discounts to the user can create the issues like consumers will find it difficult to accept the brand.

·         Other health products deliver by companies can grab the market share of Tony Ferguson in the absence of advertisements by the company.

 

Conclusion

Based on above analysis, it can be concluded that Tony Ferguson has huge opportunity available to expand its business by adopting product differentiation strategy. It has also an advantage over other competitors in terms of positive buyer behavior for its protein shake.
 

 References

Academic Pr. (2017). Social and administrative aspects of pharmacy in developing countries: present challenges and future solutions. http://public.ebookcentral.proquest.com/choice/publicfullrecord.aspx?p=5108510.

Business Jargons (2021). Micro environment. Retrieved from: https://businessjargons.com/micro-environment.html

Canster blue (2021). Tony Ferguson Weight Loss Program Review. Available at: https://www.canstarblue.com.au/health-beauty/brands/tony-ferguson-weight-loss-programs/#:~:text=The%20Tony%20Ferguson%2012%2DWeek,Practising%20Dietitian%20and%20Personal%20Trainer. [Accessed on 10th March 2021]

Claessens, M. (2015). The macro environment – six forces in the environment of a business. Retrieved from: https://marketing-insider.eu/macro-environment/

East, R., Singh, J., Wright, M., &Vanhuele, M. (2017). Consumer behaviour : applications in marketing (3rd ed.). SAGE.

Eckerle, C. (2017). Inbound Marketing: How nutrition brand Vega empowered employees to build friendships with customers. Retrieved from: https://www.marketingsherpa.com/article/case-study/Vega-friendships-with-customers

Farroq, U. (2017). Micro Environment Definition, Factors & Example. Retrieved from: https://www.marketingtutor.net/micro-environment-definition-factors-example/

Ginter, P. M., Duncan, W. J., & Swayne, L. E. (2018). Strategic management of health care organizations (Eighth, Ser. Online access with dda: askews (medicine). John Wiley & Sons.

Kellow, J., (2021). Tony Ferguson: Weight Loss Phase. Retrieved from: https://www.weightlossresources.co.uk/diet/reviews/tony-ferguson-weight-loss.htm

Kiron, M. (2019). Company background and swot analysis of forever 21 (1. Auflage). GRIN Verlag

Kostelijk, E. (2017). The influence of values on consumer behaviour : the value compass. Routledge, Taylor & Francis Group.

Kostelijk, E., & Alsem, K. J. (2020). Brand positioning : connecting marketing strategy and communications. Routledge.

Mariani, G. (2017). M & a and value creation : a swot analysis. G. Giappichelli.

Marketing.net (2020). Micro environment. Retrieved from: https://www.learnmarketing.net/microenvironment.htm

Miller, R. K., Washington, K. D., & Richard K. Miller & Associates. (2017). Consumer behavior 2017-2018 (12th ed., Ser. Rkma market research handbook series). Richard K Miller & Associates.

Miller, R. K., Washington, K. D., & Richard K. Miller & Associates. (2019). Consumer behavior 2019-2020 (13th ed., Ser. Rkma market research handbook series). Richard K Miller & Associates.

Murray, K. B. (2013). The retail value proposition : crafting unique experiences at compelling prices. University of Toronto Press.

Murray, K. B. (2016). The American retail value proposition : crafting unique experiences at compelling prices. University of Toronto Press.

Muth, N. D., & Zive, M. M. (2020). Sports nutrition for health professionals (Second). F.A. Davis Company. http://search.ebscohost.com/login.aspx?direct=true&scope=site&db=nlebk&db=nlabk&AN=2291983.

Nguyen, B., Melewar, T. C., & Hemsley-Brown, J. (Eds.). (2019). Strategic brand management in higher education (Ser. Routledge studies in marketing). Routledge. https://doi.org/10.4324/9780429029301

Osterwalder, A., Pigneur, Y., Bernarda, G., & Smith, A. (2014). Value proposition design : how to create products and services customers want (Ser. Strategyzer series). John Wiley & Sons.

Schiffman, L. G., & Wisenblit, J. (2019). Consumer behavior (Twelfth). Pearson.

Statista. (2020). Interest in nutritional trends in the UK 2019, by type. Retrieved from: https://www.statista.com/statistics/1085340/nutritional-trends-in-the-uk/

Tony Ferguson, (2021). Featured Products. Retrieved from: https://www.tonyferguson.co.za/

Winget, L. (2017). What's wrong with damn near everything! : how the collapse of core values is destroying us and how to fix it. Wiley.

 

Read More

RSK80006 Risk Management Assignment Sample

General Instructions for Assignment 3

• The Assignment 3 of RSK80006 Risk Management unit in 2021 Semester 2 (SUT-Hawthorn) is for 40% weighting.

• The deadline for submission of RSK80006 Assignment 3 is: 29th October 2021 (11:59PM)

• The questions of Assignment 3 are from Module 3 portions in Week 9 to Week 12 of Semester 2 as well as Modules 1 and 2 covered from other weeks (i.e. week 1 to week 8).

• The list of questions and break-up details of marks are in the following section (3.4).

• Upload your assignment submissions in designated submission portal under Assignments section in RSK80006 unit Canvas

• Use Microsoft WORD or Adobe PDF document file format for assignment help

• Use Harvard referencing style

• Tentatively, maximum page limit (excluding coversheet) for this Assignment 3 is 12 pages.

Question 1 of Assignment 3 (20 marks)

Consider any one recently completed or current project as your case-study background to prepare answer for this Question 1 of Assignment 3. Answer any one of the following options in

Question 1:

• Provide your assignment of risk management recommendations to achieve safety performance objectives in this project, which has some activities/ tasks with uncertainties from hazards of working in narrow confined underground site operations

• Provide your assignment of risk management recommendations to achieve schedule (i.e. time related) performance objectives in this project that encounters uncertainties such as delays in some materials supply, contractor disputes, significant changes/ variations, and some adverse weather conditions during project delivery periods.

• Provide your assignment of risk management recommendations to achieve quality performance objectives in this project, which has noteworthy observations of rework, wastages, defects, and some non-compliances rendering quality deviations.

• Provide your assignment of environmental risk assessment and risk management recommendations to achieve environmental/ sustainability performance objectives in this project, which encounters some environmental impact assessments for significant levels of noise, dust, and hazardous chemicals found in project operations.

Question 2 of Assignment 3 (20 marks)

Assume that you are responsible for risk management in a particular enterprise/ organization in an engineering industry. You are required to prepare your assignment for designing and implementing a risk management system in your organization. Your assignment need to consider the following:

• Risks identified, targeted objectives and values
• Risk criteria and management agenda
• Design details of risk management system including roles and responsibilities for risk management
• Implementation Plan for the risk management system
• Monitoring and reviewing/ auditing plan for the risk management system

Solution

Question 1: Risk management recommendations to achieve safety performance objectives

Introduction

UGL is one of the leading asset solution providers that operate in many different sectors such as engineering design, asset management, operation and maintenance, facilities handling, and manufacturing sectors (Ugllimited. 2021). The company is working on the West Angelas Power Station, located in West Australia. This power station is integral to the mining network in Pilbara, West Australia. The power station is responsible for providing power to the rail network that connects different parts of the mining network. UGL is tasked with finishing the construction and installation of equipment for the power station and managing various aspects of the overall project. Construction processes are known for containing high-risk factors (Valipour et al. 2017). This report attempts to assess the safety risks that can impact the project completion and to provide recommendations that can help the project completion successful regarding sustainability and environmental goals. The assessment is provided in a tabular form with the help of well-described risk criteria and risk scores. Risk scores can enable defining the proper weightage of each risk factor. This is done via numerical balancing based on priority.

Risks involved in the project

The completion of the construction and installation project is highly complex. The company UGL has to uphold the complete responsibility of managing the entire project. It is essential to effectively handle risks in construction processes (Serpell, Ferrada, and Rubio, 2017). Proper handling of such risks can be used to improve the performance of a company (Masuin, Latief, and Zagloel, 2020). Sustainability is one of the main requirements of intricate construction processes (Ahn, Kim, and Kim, 2020). Unique risks and Obstacles can come up in the completion of the process. This section of the report will make sure that the safety risks that are involved in the project are adequately assessed. The installation of two generators that are essential to the functions of the power station was one of the most high-risk operations (Ugllimited. 2021). The company also had to deal with the consumer gas system, diesel inventories, and fire detection ((Ugllimited. 2021)). There are various risks associated with the installation of such types of equipment. The workers that are in charge of on-field activities run the risk of getting exposed to harmful gas, inhaling diesel and other harmful substances, getting burned through power leaks, and electric shock due to malfunctioning equipment. The construction site of the project contains risks such as the collapse of building parts, injury due to mishandling of machinery, hearing loss due to loud noises. The transport and logistics risks are also present due to UGL handling the procurement process of materials.

Risk Assessment

Recommendations

Some suggestions can be made after analysing the construction process taken up by UGL. These recommendations can be implemented to significantly reduce the number of mishaps that can happen on the construction site. Suggestions made in the report should be argued between experienced workers (Borkovskaya, Bardenwerper, and Roe, 2018). These suggestions are provided below-

• Providing protective gear to personnel who are working on the construction site is recommended in this regard. Gears such as masks, gloves, safety boots can be used to safeguard individuals from any mishaps on the site.

• Upgrading existing pieces of equipment and purchasing new pieces of equipment that are more suited to handle the complex operations of a construction site is recommended. Modern technologies and pieces of equipment have implemented many safety mechanisms that are non-existent in previous models. This is why upgrading equipment is necessary.

• The company must implement a supervisory team that will audit and monitor the workers and ensure that they are Abiding by the safety regulations. Interventions can also work in refining safety conduct (Zaira and Hadikusumo, 2017). Teams of specialists and technicians should also be created that will check and review the status of the pieces of machinery that are used onsite.

• Arranging workshops and training programs can be beneficial to make sure that the workers are adept at handling complex equipment.

Conclusion

This section marks the end of the safety assessment report of the construction project of the West Angelas Power Station, executed by UGL. This report takes a look at different risks that are associated with the many different operations and activities executed by UGL throughout the construction process. The risks are outlined and assessed in a risk matrix. Adequate risk scores are provided so that the risks can be sorted based on their impact on its operations. Recommendations that can help the company avoid and exterminate these risks are also provided in the report.

Question 2: Risk management for Lendlease

Introduction

Risk management and planning are significant organizational functions that help ensure that the organization finds success in its operations. It is crucial to implement knowledge and planning regarding risk in choices made by businesses (Vij, 2019). Among others, companies working in the engineering industry also need to apply proper risk planning and management procedures. The planning phase highlights the crucially of actions and guides communication regarding these actions (Willumsen et al. 2019). This report attempts to create a proper risk management system for a prominent construction company, Lendlease. It is an Australian company operating in the real estate industry headquartered in Sydney, Australia (Lendlease. 2021). The report will provide an in-depth analysis of the risks that are apparent in Lendlease’s operations, the goals that need to be met through the risk mitigation system, and how the company’s values align with it. It will contain relevant standards and agendas. The design of the risk management system will be outlined along with a fleshed-out audit plan that will help the company monitor the progress and the system’s functions. The report will conclude with a holistic overview of the entire process.

Risks identified, targeted objectives and values.

Risk identification and highlighting requires a holistic analysis of the operations of Lendlease and recognizing the possible obstructions that can be potentially damaging to the standards, expenses, construction time, and the scope of the company operations. Risk identification generally is executed with groupings depending on their nature or without any groupings attempts (Siraj and Fayek, 2019). Recognition of risks and managing them remains some of the highest priorities of construction projects (Chattapadhyay and Putta, 2021). This is the reason why a proper risk identification of Lendlease's construction operations is needed. The main risks that are prominent in the construction industry can be classified into three separate groups. These are strategic risks, operational risks, and economic risks. Strategic risks are roadblocks that obstruct specific business tactics in achieving success. Operational risks are risks that arise because of incompetency and failure of certain operations, and economic risks are factors that affect the expenditure and revenue of the company. The company is struggling to handle economic risks, as evidenced by the financial struggles of the company in 2019 (Hayford, 2020). Many risks can pop up in Lendlease’s operations.

• The company is a multinational. This means that a sudden shift in the political landscape can pose a threat to the company's operations. The company is explicitly concerned about the US elections, uncertainties regarding Brexit, and the Italian political scenario (Marsh. 2021).

• Unemployment is also a significant concern for multinationals. These risks, along with safety hazards, Natural disasters, contract issues, procurement problems, and social issues such as involuntary migration occurring in many countries, can be identified as major risks for the company.

The main objective of the risk management system will be to make sure that the clients, suppliers, owners, and other stakeholders of the company can attain their goals from the operations of Lendlease and lower negative effects of identified factors on the expenditure, time consumption, and quality of the company's operations. The values of the company align with the risk management systems as it enables the comp [any to fulfil its promises to its stakeholders and maximize value creation for the shareholders.

Risks criteria and management agenda

The criteria are the general standards of risks that will be defined in this segment of the report. It is vital to set up points of reference for the risk level to make it easier to assess certain types of risks and their severities. These criteria can be used for the assessment of several types of risk because of their general nature. Five different criteria have been defined for this particular report. In ascending order of severity, these criteria are – negligible, minor, moderate, significant severe. The utilization of These criteria, along with the likelihood of occurrence of a specific risk, can help assess the importance of a particular risk factor. This can help in prioritizing which risks are to be eliminated first. The likelihood of occurrence has been classified into five subgroups. In descending order of occurrence, these groups are very likely, likely, possible, unlikely, and very unlikely. A risk score is assigned based on these criteria. Description of each criterion, risk score, and other factors are provided in tabular formats.

Design details of the risks management system

This section of the report will outline the details in consideration for the design of the risk management system. The company should be able to apply the risk management system in all operations in the future. This is the reason why the designing process has to be general and constructed in such a way that it can be used in different circumstances. Every organization is dealing with its specific risks, and the risks change drastically based on the operations and roles of the organization (Abd Samad and Dahlan, 2019). For specific problems to not occur, risk management systems should be customized to and complement the specific organization it is being applied in (Willumsen, Oehmen, and Rossi, 2019). Identification of the potential risks is the first step of the system. Before the start of a new operation, experts will assess the potential risks to make sure that the operation is not stunted in the future. Analysis and evaluation of the identified risks will be the second step of the system. Analysts will examine the identified risks and categories them. In this step, the risk criteria and risk scores will be utilized. Step three is the treatment plan. In this stage, actions and strategies will be implemented to avoid, diminish and mitigate the risks that are identified and evaluated. Step four is reviewing the risk management system in light of the insights gained through tackling said project's risks. This review process will add new value to the system making it well equipped to handle risks in the future. Step five of the design process will pay attention to strategies that can help avoid similar risks in the future.
Implementation plan

The implementation plan guides the incorporation of specific strategies and tactics that can help Lendlease with its future projects. This plan will outline the responsibilities and duties that will be given to certain workers, along with pointing out the authorities that will be in charge of the risk management process. This plan will also contain information regarding the procurement and storing of information that is needed to run the project smoothly. Designing and implementation of communication methods that let the operation team quickly convey information to each other is also a part of the implementation plan. The implementation plan and its incorporation are crucial to the success of any construction activities. Various risks that have not been estimated can pop up during the execution of a particular operation. Lendlease must develop accompanying contingency plans that will help in dealing with these risks. The implementation plan will follow the health and safety regulations decreed by the government of the specific nation that the company is working to diminish chances of injury of workers and any other damages that can be resulted from unassessed factors. Steps will be highlighted in the implementation plan that can help with recovering from unexpected disasters that can occur in a construction operation.

Monitoring

Continuous monitoring can help in making sure that the risk management system is working smoothly and individuals are upholding the duties that are reallocated to them. Auditing and review can help with strengthening the system and making it equipped to face the ever-changing risks of the construction industry. Reviewing and updating the system with insights gained from operations can be suggested in this regard to Lendlease. While reviewing the system, the company should take into account feedback from stakeholders such as individuals working on sight, shareholders, project managers, and others who have a significant interest in the projects taken and operations executed by Lendlease. Individuals should be assigned to the task of effective monitoring of the operations taken by Lendlease. Reviews that are monthly, bi-annually, and annually executed are recommended to the company by this report.

Conclusion

This report reports on the creation of a risk management system at Lendlease to aid their operations. The report identifies the risks that are associated with the organizational operations and recommends detail that can help in the creation of the company's risk management plan. Recommendations regarding a prototype agenda, risk criteria formation, and monitoring of the said system are also provided in the report.

References

Abd Samad, W.A.B.W. and Dahlan, A.R.A., 2019. An Integrated Risk Management using Business Model Canvas: Designing Business Model Options for The Royal Malaysian Air Force in The Digital Era. http://eprints.utm.my/id/eprint/view/subjects/T1.html

Ahn, S., Kim, T. and Kim, J.M., 2020. Sustainable Risk Assessment through the Analysis of Financial Losses from Third-Party Damage in Bridge Construction. Sustainability, 12(8), p.3435. https://doi.org/10.3390/su12083435

Borkovskaya, V., Bardenwerper, W. and Roe, R., 2018, June. Interactive teaching of risk management in the Russian construction industry. In IOP Conference Series: Materials Science and Engineering (Vol. 365, No. 6, p. 062030). IOP Publishing. https://iopscience.iop.org/article/10.1088/1757-899X/365/6/062030/meta

Chattapadhyay, D.B. and Putta, J., 2021. Risk Identification, Assessments, and Prediction for Mega Construction Projects: A Risk Prediction Paradigm Based on Cross Analytical-Machine Learning Model. Buildings, 11(4), p.172. https://doi.org/10.3390/buildings11040172

Hayford, O., 2020. Rebalancing risk allocation on infrastructure projects. Built Environment Economist: Australia and New Zealand, (Sep-Nov 2020), pp.4-8. https://search.informit.org/doi/pdf/10.3316/informit.524238664975909
Lendlease. 2021. About us. [online] Available at: <https://www.lendlease.com/company/about-us/> [Accessed 19 October 2021].

Marsh. 2021. Lendlease: Risk the Heartbeat of the Business. [online] Available at: <https://www.marsh.com/content/marsh/pacific/au/en_au/insights/research/lendlease-risk-the-heartbeat-of-the-business.html> [Accessed 19 October 2021].

Masen, R., Latief, Y. and Zagloel, T.Y., 2020. Development of integration risk on integrated management system in order to increase organizational performance of construction company. International Journal of Project Organization and Management, 12(2), pp.164-177. https://www.inderscienceonline.com/doi/abs/10.1504/IJPOM.2020.106379

Serpell, A., Ferrada, X. and Rubio, N.L., 2017. Fostering the effective usage of risk management in construction. Journal of Civil Engineering and Management, 23(7), pp.858-867. https://doi.org/10.3846/13923730.2017.1321578

Siraj, N.B. and Fayek, A.R., 2019. Risk identification and common risks in construction: Literature review and content analysis. Journal of Construction Engineering and Management, 145(9), p.03119004. https://ascelibrary.org/doi/abs/10.1061/(ASCE)CO.1943-7862.0001685

Ugllimited. 2021. Our approach. [online] Available at: <https://www.ugllimited.com/en/about-us/our-company/our-approach> [Accessed 19 October 2021].

Valipour, A., Yahaya, N., Md Noor, N., Antuchevi?ien?, J. and Tamošaitien?, J., 2017. Hybrid SWARA-COPRAS method for risk assessment in deep foundation excavation project: An Iranian case study. Journal of civil engineering and management, 23(4), pp.524-532. https://doi.org/10.3846/13923730.2017.1281842

Vij, M., 2019. The emerging importance of risk management and enterprise risk management strategies in the Indian hospitality industry: Senior managements’ perspective. Worldwide Hospitality and Tourism Themes. https://doi.org/10.1108/WHATT-04-2019-0023

Willumsen, P., Oehmen, J. and Rossi, M., 2019, July. Designing risk management: Applying value stream mapping to risk management. In Proceedings of the Design Society: International Conference on Engineering Design (Vol. 1, No. 1, pp. 2229-2238). Cambridge University Press. https://www.cambridge.org/core/journals/proceedings-of-the-international-conference-on-engineering-design/article/designing-risk-management-applying-value-stream-mapping-to-risk-management/B0BF622528D26D6DECE8C8D757DD9967

Willumsen, P., Oehmen, J., Stingl, V. and Geraldi, J., 2019. Value creation through project risk management. International Journal of Project Management, 37(5), pp.731-749. https://doi.org/10.1016/j.ijproman.2019.01.007

Zaira, M.M. and Hadikusumo, B.H., 2017. Structural equation model of integrated safety intervention practices affecting the safety behavior of workers in the construction industry. Safety science, 98, pp.124-135. https://doi.org/10.1016/j.ssci.2017.06.007

Read More

ENV101 Introduction To Events Assignment Sample

Individual/Group - Group (2-3)
Length - 1500 words (+/- 10%)

Learning Outcomes

The Subject Learning Outcomes demonstrated by successful completion of the task below include:

a) Research and explore the characteristics of a particular event from any industry.
b) Outline the stages involved in the planning and implementation of events
c) Identify and describe the various tasks involved in event production.

Submission - Due by 11:55pm AEST/AEDT Sunday end of Module 6.1 (Week 11).

Weighting - 35 %
Total Marks - 100 marks

Task Instructions for Assignment Help

To complete this task you will create an event manual outline based on an event that you will create. The event is a family fun day to be held in Brisbane, Queensland. The theme is “Our Nation Family Fun Day”, which intends to celebrate multiculturalism. It will feature a stage show and a range of activities of your choosing, suitable for all age groups.

Event details

Name: Our Nation Family Fun Day
Date: The last weekend of February – the end of the Australian summer (insert a day and date).
Location: Cultural Forecourt, South Bank Parklands, Brisbane.
Time: The event will run from 9:00am to 9:00pm.

Description: Your event will be a family fun day celebrating the many different nationalities and cultural origins in our society.

Audience: All age groups

Solution

1.0 About this manual

The primary purpose of this event manual is to provide references and guidelines to every staff and all the other advisors regarding the organised event (Krumeich, 2014).

2.0 Event details

2.1 Explanation of the event detail at the front of the manual

An event manual consists of various essential data regarding the event. The event organisers are the ones who wrote the event manual document. An event manual ensures an overall description to the staff and other sponsors and stakeholders about the event operation (Dowson & Bassett, 2018). The event manual logged every date, time and information in one document as it help the working staffs and other team members make key decisions and biome aware of the overall work in the event. In an event manual, the information seems quite extensive and therefore to get a glance at the event manual's detail needs to be printed on the front page. The description on the front page also depends on the complexity of the event. The distribution of the event manual also needs to be held only to the key individuals.

2.2 Description of the created event

Name: Our Nation Family Fun Day.

Date: 27th Feb, Sunday.

Location: Cultural Forecourt, South Bank Parklands, Brisbane.

Time: The event will start at 9.00 am and will end at 9.00 pm.

Description: The theme of the event is based on multiculturalism. Primarily the event will focus on celebrating multiculturalism in the presence of the multicultural family.

Audience: The event welcomes every individual of all age groups.

3.0 Program of events

3.1 Importance of the programme of events

The programme of events is an essential component for the success of the event. An event programme is a helpful tool that ensures effective planning. The programme of event also ensures that whether any event is missed. The programme of events specifically lists each and every programme with a proper timeframe (Van der Wagen & White, 2018). It helps the audience and also the other staff to know the activity timeline of the event. The programme of events also conveys the message to the target audience that the event organiser wants to deliver. A well-prepared programme of event flyers or posters also keeps the audience of the programme interested in the upcoming activities of the event. The programme of the event is also important because it keeps the flow of the event running for the audience. However, the main importance of the programme of events poster is to keep the event at its current time (Dashper et al., 2018). Time management becomes easier after handing out the flyers or the poster for the event management organisation. The event programme posters also provided the chronological order of the activities in the event. It helps the performers also to know about their performance time and prepare their activities according to the mentioned time.

3.2 Programme of the event


Table 1: Programme of event Timeline and description

3.3 Programme of event flyers

Figure 1: Flyers for the Event

4.0 Site map

4.1 Importance of site map

Creating a site map for the audience is extremely vital for the event management organiser. The site map represents a visual direction for the audience (Jackson et al., 2018). Apart from this, the site map also helps to handle the flow of the people better. The site map also helps the audience to know which programme is conducted at which place. The primary importance of the site map is in the event navigation. In the case of the event staff, the site map makes it easier for the staff to navigate the flow of the attendees through the event timeline. The event organiser also uses the site plan as a part of their planning process (Bladen et al., 2018). The site pan can also be very valuable in case of any emergency. It shows the path of any emergency exit to the audience.

4.2 Site P lan of ‘Our Nation Family Fun Day’ event


Figure 2: Site Plan of ‘Our Nation Family Fun Day’

 

5.0 Staff allocations

5.1 Importance of staff allocation

Every member of the committee, staff, and volunteers are essential human resources for any event to be successful. Staff allocation always left a great impact on the event’s success. Staff play a significant role in event management. Staff allocation brought efficient performance in all other functions (Allen, 2019). Staff allocation is the key to managing all other functions to work swiftly. The event controller needs to recruit competent personnel for every job that further helps the event to run very smoothly. The staff allocation also needs to be according to the personnel's technological ability (Van Niekerk & Getz 2019). In any event, the new technology helps the event staff work easy and accurate. In the case of allocating the staff in the perfect role, the event controller should allocate staff where the person can utilise the technology for better management (RAJ et al., 2022). The event controller plays the key role in the staff allocation and provides the role for the staff according to their capability.

5.2 Allocation of staff at ‘Our Nation Family Fun Day’

Table 2: Staff Allocation

6.0 Security accreditation

6.1 Importance of Security accreditation

Security accreditation is one of the key essential for the event to be successful. In any kind of event, the accreditation ensures where the person should be at the right palace and right time. A security event accreditation provides multiple functionalities (Silvers & O'Toole, 2021). These include the identifications, access, per diems and overall security. Not every area of the event space is accessible for every person. Therefore, security accreditation helps the security personnel to watch which people have the access to which particular areas.

6.2 Category of accreditation

Table 3: Category of accreditation
Source: Self-developed

7.0 References

Read More

TME6014 Technology Management and Entrepreneurship Assignment Sample

The Spending.csv is a data set collected by a direct marketer who sells his products only via direct mail. He sends catalogs with product characteristics to customers who then order directly from the catalogs. The data set covers 1000 customers and the following attributes of each customer are collected

i. Age (of customer; old/middle/young)
ii. Gender (male/female)
iii. OwnHome (whether customer owns home)
iv. Married (single/married)
v. Location (far/close; in terms of distance to the nearest brick and mortar store
that sells similar products);
vi. Salary (yearly salary of customer; in dollars);
vii. Children (number of children);
viii. History (of previous purchase volume; low/medium/high/NA; NA means that
this customer has not yet purchased);
ix. Catalogs (number of catalogs sent);
x. AmountSpent (in dollars).

Need to complete these tasks for assignment help -

1 -Find if there is there any linear relationship (positive/negative) between any one of following attributes (Age, Gender, OwnHome, Location, Salary, Children, History, Catalogs) and the AmountSpent attribute? If there is any, explain it with a graph visualization.

2 - Explore the dataset thoroughly using graphs and bring the following findings in front of marketer using graphs. Explain it in the form of a report.

a. Does age matters in the overall spending
b. Do different genders spend different amount?
c. Does having house matters in spendings?
d. Does being married matters in amount-spent?
e. Does location matter in amount-spent?
f. Does salary matter in amount-spent?
g. How having children matters in amount spending?
h. Does catalogs have an effect on amount spending.

3- Write at least 10 recommendations for marketers to make strategy to increase his overall revenue.

Solution

Question1

The correlation test is used to determine the relationship between the two variables (Mat Roni, Merga, and Morris,2020) The relationship between the amount spent and the salary of the individuals has been identified with help of a correlation test. It has been found from the correlation analysis that a positive relationship exists between the two variables namely the amount spent and the salary. Here the dependent variable is the amount spent and the salary of the individuals is regarded as the independent variable. It has been identified from the results of the correlation test that a positive correlation exists between the two amounts spent and the salary of the individuals. The positive correlation in this context is 0.69. Thus it can be concluded that with the increase in the salary of the individuals considered in the survey, their expenditure gradually increases. The result of the correlation test is given below


Figure.1-Representing the correlation between the amount spent and the salary
(Source: Self –Developed)

In the Figure-1, the relationship between the amount spent and salary is depicted .This suggests that a positive correlation relationship occurs between the amount spent and the salary.This suggests that with the increase in the salary of the individuals, the expenditure gets increased.

Question2

Relation between age and expenditure

Relation between gender and the expenditure

Relation between owning a house and expenditure

Relation between marriage and expenditure

Relation between the location and the amount spent

Relation between salary and expenditure

A positive correlation has been observed in the case of the salary and the amount spent. The explanation is given above


Relation between number of children and expenditure

A negative correlation of -0.23 has been observed between the amount spent and the number of children. This implies that with an increase in the number of children, the expenditure of the individual decreases.

Relation between the number of children and the amount spent

A positive correlation of 0.45 has been observed between the amount spent and the number of catalogues. This means with the increase in the catalogue number, the amount spent by the individual increases.

Question-3

The recommendations that should be applied for the improving the revenue of the organization

1. Goal development-The goals must be set in order to develop the revenue of the company. Many improvised plans should be implemented keeping in mind the previous revenue record of the company( Gallo, Sosa and Velez-Calle,2022).The appropriate methods should be chosen for the development of the improved revenue goals.

2. Aiming for the regular customers-The market could specifically target the customers who frequently purchase the products. The marketer could reach out to them by means of email or messaging. Regular customers should be encouraged to purchase more which could in turn help in enhancing the overall revenue (Vaní?ková,2018)

3. Aiming for former customers- In in some cases the customers often refrain from buying from a particular marketer for some reason. This reason has to be properly identified by the marketer. The marketer should try to bring back the former customers. This can be done by reaching them through frequent emails or telephone conversations.

4. Enhancement of geographic expansion-The marketer could try to expand their customer base over a wide range of the region. This would help in popularizing the organization over a wide range. This would ultimately help in the enhancement of the customer base of the organization.

5. The reformation of the product pricing-The pricing of the commodities has to be adjusted keeping in mind the demands of the market and the existing price of the products that are being sold by the marketer. An attempt should be made to reduce the price of the product which would enhance the revenue of the organization.

6. Improvisation of services or products-The products should be improvised keeping in mind the demand of the customers. New products could be introduced to attract the customer base. Moreover, the introduction of new services like a free home delivery system can also be introduced for increasing the revenue of the organization

7. Introduction of new payment modes-The new payment modes should be introduced like various online payment modes should be encouraged since digital money has become quite popular these days.

8. Encouraging delayed payments-The customers can be encouraged to buy the products and pay later on as per their convenience. This could help in popularizing the organization among the customers.

9. Introduction of subscription services-The subscription services must be encouraged and the customers should be made aware of the availability of the new products keeping in mind the demand of the market.

10. Providing offers and discount-The customers should be provided with appropriate offers and discount services for popularizing the products among the customers. The sale of products at a discounted price could help in enhancing the revenue of the organization (Miao, Wang, and Zha, 2020).

References

Read More

MANU2117 Integrated logistic support Management Assignment Sample

Question 1.

a. Select a logistics/transportation or manufacturing system of your choice and describe the applicable life-cycle phases and activities, tailoring your description to that system.

b. As best you can, identify life-cycle activities that occur in the waterfall model, the spiral model, and the “vee” model. Of these models, pick the one you prefer for your selected model and explain why.

Question 2.

Identify a specific logistics/transportation or manufacturing problem that you wish to solve through the design and development of a new system (must select a different problem from Question 1 above problem). For your system: (2 points for each part)

a. Describe the current deficiency and identify the need for the new system.
b. Discuss the various alternative technical approaches that you may wish to consider in designing the new system.
c. Define the basic operational requirements for the new system.
d. For the selected problem, develop operational functional flow block diagram to the third level.

Question 3.

Logistics system includes those functions, associated primarily with the forward flow of activities illustrated in the figure below, involved with the initial acquisition (procurement) of items from various sources of supply, the flow of materials throughout the production/ construction process, the transportation and distribution of products from the manufacturer to the customer (user), the sustaining on-site customer service as required, and all the related business-oriented processes (e.g., accounting, financial management, information flow, money flow) necessary to ensure that the entire flow is effective and efficient. A simplified illustration of this flow is presented in the figure below. The figure shows different paths that may be utilized depending on the requirements; that is, materials shipped from vendor or supplier to manufacturer, manufacturer to warehouses, warehouses to end-users or customers and so on. Given a top-level definition of the logistics system through the functional analysis, the system has been broken down into components or subsystems and lower-level elements such as raw material suppliers (vendors), manufacturers, warehouses, end-users or customers, transportation system, and so on. The challenge is to identify individual functional requirements and associated resources on an independent basis. Consider the following data are available for designing the logistics system:

Product: Motor Generator for industrial use, Total annual demand = 5,500 units of motor generator, Production capacity of a single manufacturing plant = 2,000 units per year (maximum capacity), Potential Location of Manufacturing plants = in Asia (e.g. China, India, Thailand, Vietnam, etc.), Potential Demand Centres are based in Western Europe and North America (USA, Canada and Mexico), Potential Location of Vendors or Raw Material Suppliers are in Asia and North America, Transportation Systems used to transport raw materials, finished product (generators) = trucks, rail line, ships (waterways), Loading and unloading machines used at Warehouses = Fork-Lift machines, conveyors (heavy duty), Total number of end-users or customers (or demand centres) = 5,

Maximum Warehouse capacity to hold items (generators) = 1,500 per warehouse per year,
Maximum capacity of trucks to transport generators = 25 generators per truck per trip,
Maximum capacity of rail line = 150 generators per trip,
Maximum capacity of cargo ships = 1,000 generators per trip,

Volume of raw materials consumption per unit of generator manufacturing = 3* volume of a generator, Raw materials are carried using the similar modes of transportation, but no warehousing is required, Transportation Cost from raw material supplier to Manufacturer = $3.5 per generator volume by truck, $2.0 per generator volume by rail line, and $1.25 per generator volume by cargo ships, Transportation Cost from Manufacturer to Warehouses = $4.0 per generator by truck, $3.0 per generator by rail line, and $1.50 per generator by cargo ships, Transportation Cost from Warehouses to end-user or customers = $3.0 per generator by truck, $2.0 per generator by rail line, and there are no waterways for cargo ship transport, Assume that the assembly line equipment requirement at the manufacturing plants, raw-material suppliers’ sites, and warehouses have already been designed and installed and you do not need to calculate it. Complete the requirement analysis for the above logistics system and determine the resources (both equipment and manpower) required. Analyze it using the Concepts and ideas covered in the class lectures and relevant chapters in the textbook and reference book. Use as many notes, references and case studies as you can get hold of and need to. You can make reasonable assumption for any missing data, but you need to use the given data in the calculations (if data is given above).

Question 4

Cairns Airport is one of Australia's leading regional airports, providing both international and domestic flights while being the gateway to Northern Australia. It is the seventh busiest airport in Australia. The airport is located 4.3 km northwest of Cairns or 7 kilometres north of the Cairns central business district, in the suburb of Aeroglen.

At Cairns airport in Queensland, Australia, it takes on the average 5 minutes to land an aero-plane, once it is given the signal to land. The landing time is exponentially distributed. Although incoming planes have scheduled arrival times, the wide variability in arrival times produces an effect which makes the incoming planes appear to arrive in a Poisson fashion at an average rate of 6 per hour. This produces occasional stack-ups at the airport which can be dangerous and very costly. Under these circumstances, how much time will a pilot expect to spend circling the field waiting to land? If the aviation controller wants to reduce the maximum time spent circling the field waiting to land to 5 minutes, what is the optimal recommendation?

After landing at the airport, all international passengers walk on a travellator to arrive at the Immigration counters for immigration clearance. The average arrival rate of the passengers at the immigration counters is 1.2 per minute and the arrival process is assumed to be Poisson distributed. There are five immigration service counters open at any given time at the international terminal of the Cairns airport and each officer can serve 16 passengers per hour with service time exponentially distributed. Determine the average queue length waiting time of the passengers in the queue in front of the immigration. If the expected waiting time is required to be less than or equal to 5 minutes, what changes are needed in terms of number of immigration counters and the design.

Following the immigration clearance, passengers are directed to Baggage Claim area of the airport. Assuming the arrival rate of the passengers at the Baggage Claim to be equal to overall service rate at the immigration counters and all the passengers have checked-in baggage to claim, and average time to search and collect the baggage from any of the two identical moving conveyors to be 7 minutes which is exponentially distributed. Determine the optimal number of moving conveyors required at the international terminal if the maximum waiting time should not exceed 12 minutes at the Baggage Claim conveyors. After the baggage claim, the passengers are directed to Customs and Quarantine area of the airport. Suppose 20 per cent of the passengers who claim their baggage do not have any item to declare at the Customs and Quarantine inspection and exit through the green signal channel and proceed to Taxi stand of the airport. The remaining 80% of the arriving passengers at the baggage Claim exit are directed to Quarantine area equipped with five X-Ray machines and inspectors. On the average passengers spend 10 minutes at the X-Ray machines and baggage inspection before they leave for Taxi stand. If you happen to be one of passengers, give your recommendation for the optimal number of X-Ray machines and inspectors. Comment on the existing facility design.

Question 5

At the Geelong port in Victoria there are twelve berths and seven unloading crews. When all the berths are full, arriving ships are diverted to an overflow facility 30 kilometres down the shore in Docklands port area near Melbourne city. Oil tankers from the Middle-East region’s oilfields arrive according to a Poisson process with a mean of every 65 minutes. It takes an unloading crew, on the average, six hours to unload a tanker, the unloading time following an exponential distribution.

a). On the average, how many tankers are at the port?
b). On the average, how long does a tanker spend at the port?
c). What is the average arrival rate at the overflow facility?
d). What is the probability that the unloading crews will be sitting idle?

Determine the optimal number of unloading crews if the captain of the ship cannot spend longer than 8 hours at the Melbourne Port due to his other commitments at Sydney and Brisbane ports.

Solution

Question 1

The procedures required in transferring materials and goods along the supply chain are subsequently referred to as logistics. Distribution and transportation systems are components of the logistic system. Transport is the process of using transportation to obtain raw materials, handle materials, and distributing goods from the production house to the using point. Systems for supply chains include and are subsets of logistics. The supply chain is basically a process of obtaining and moving components and raw materials, moving items in and out of manufacturing facilities, storing, loading and unloading goods, and delivering goods to clients. Some companies provide a GPS tracking system to their customers. This system helps the customers by allowing them to track the location of their product. According to studies, this system is more liable as the customers can monitor the status of delivery for their products. DB Schenker is one of the leading logistic companies in Australia.

a) In order to keep logistics flow, DB Schenker offers customers in a wide range of sectors logistics operations across the entire supply chain, including planning, acquiring, production, storage, and delivery as well as returns transportation and valuation services. The increase and fall in consumer demand for technology is influenced by seasonality, advertising and promotion activities, product life cycles, and the intensity of competition. The distribution network frequently has to manage erratic volume swings from manufacture through delivery. Rising demand in new countries necessitates value solutions, labour, and local knowledge.
Life cycle of a product means the whole procedure of the development of the product. The prototype design can be considered to be the starting point of product development. This cycle also involves the improvement which was done to the product during the research. Employees may be in danger if the dock equipment is improperly chosen (Lenget al. 2020). To meet customer demands, logistic companies should also have strong storing units which can help in storing their goods during the production time. Life cycle phases and activities of DB Schenker is planning, organizational improvement, implementation, processing of order, updating the inventory, distributing goods, receiving goods and acknowledging all the transactions. The phases of the logistic company are intended to gather all the other uses of organization that includes the sustainment, disposal and acquisitions of different systems. The key objective of the life cycle of DB Schenker is to make sure that the sustainment is the basic consideration of all the activities which are linked with the phases of the system beyond the life cycle. As a result, it must be ensured that the least strategy of cost is being used to maximize the outcome over time.

Figure 1: Life cycle phases of logistics company
(Source: Self-made)

b) Vee model for assignment help is found to be suitable for DB Schenker as the product needs to be delivered from the manufacturer to the customer with proper management system. The appearance of the product, the information about the development of the product, the procedure of delivery and shipment, and the roles of the concerned team. Vee model has a higher amount of success over the waterfall model which can bring sustainability to the DB Schenker. The company should start using this model as it is providing a detailed life cycle model. As the world is advancing towards a technological era, these companies should also make it a part of their organisation. Technologies like a delivery tracking system, Data monitoring systems and scheduled delivery systems should be included in their program.


Question 2

One of the leading providers of supply chain solutions is Kuehne + Nagel International AG (K+N). The company's primary line of business is the supply of end-to-end logistics solutions, including contract logistics and integrated logistics for rail, road, air, sea, and other modes of transportation. Additionally, the business concentrates on offering IT-based logistics solutions. Aerospace, automobile, elevated, retailing, fast-moving basic goods, pharmaceutical and healthcare, industrial, and oil & gas are some of the key sectors the company serves. It also serves as a strategic solutions company, including fairs, events, and emergency and relief logistics. The Management of Kuehne + Nagel International logistics of physical products needs the integration of some factors, including information flow, material handling, product production, product packaging, storage control, and transportation to the consumer. The problem is the "card punching attendance system" is not proper in the company. A new system of attendance which is strong enough to monitor the attendance of the company must be designed to ignore all the significant losses of the company.

a) The current deficiency in the "card punching attendance system" should be designed in such a manner that helps in the personal approach of all the employees in case of their absence. The card punching might be carried out by other workers. However, the new system is required to avoid the unethical manner of card punching and provided work should have the capacity to make their attendance in the system card punching by the employees.

The company started joint ventures with other tech giants to resolve this problem. However, their main problem still lies in the attendance system. The lack in this aspect creates an opportunity for the rival company (Malindzakova and Zimon, 2019). So, to increase their sales, they need to work on their weakness and find some innovative and time-saving options. The company should combine their mid-capacity vehicles with low-capacity vehicles. This will create a desired change in the supply chain. The supply chain should be rebuilt and the number of products which are needed to be delivered should be monitored.

b) Many technical approaches are required to be considered when the person desires to design the new system. Firstly, it is required to use the camera for monitoring the unethical manner of card punching and secondly, the "biometric finger punching" which is probably found in the attendance system.

c) There are several basic operational requirements of the biometric new system. First, the system of attendance should have the capacity to have the entire management functionality of leave. Secondly, the system must have the capability to configure the changes within the figures of the employees (Winkelhaus and Grosse, 2020).

The new system must cover all the weak points of the company. The need for a new warehouse and good quality vehicles should be fulfilled as soon as possible. To tackle this problem, the company needs to work on its weak points. The basic operation starts from changing the goods carrying capacity of the vehicles. Vehicles are warehouses are the main component of a logistic company. So, keeping a good warehouse along with a wide range of vehicles is a necessity for the company.

d) The company should start using new technologies for their transportation. Drones are deployed for an endless variety of functions and seem to be everywhere.

Figure 2: Operational Functional block diagram
(Source: Self-developed)


Question 3

The process of using transportation to gather raw materials, handle commodities and distribute goods from the manufacturing unit to the customer. Supply chains are considered to be the key part of the system. Obtaining and moving raw materials, storing, loading, and unloading goods and transporting the finished goods to customers make up the supply chain (Giusti et al. 2019). As stated above, if the supply chain problem is resolved then the company will tackle its biggest problem which will help the company in regaining its customers. Logistics managers are essential in the current status of our global industry. Their role is very crucial for businesses as they transport their goods from the source to the market.

The equipment of the dock has a high significance. Employees may be in danger if the dock equipment is improperly chosen. Since the docking area is where the shipping and receiving processes converge, safety there should always come first (Facchiniet al. 2019).
The requirement of a motor generator: 5500 units

Maximum output of a single production plant: 2,000 units per units

Potential Demand Centres are headquartered in Western Europe and North America (USA, Canada, and Mexico), Potential Locations of Vendors or Raw Material Suppliers are in Asia and North America, and Potential Locations of Manufacturing Plants Are in Asia. Vehicles, railroads, and ships are examples of transportation systems used to move raw materials and finished goods.

Machines for loading and unloading used in warehouses are heavy-duty conveyors and Folk-lift machines.
Number of customers:5
Warehouse maximum capacity to hold goods:1500 per warehouse
The maximum number of generators that can be supplied by trucks: is 25 generators per truck
Rail line capacity: 150 generators
Max capacity of cargo ships: 1,000 generators per trip
The volume of raw materials consumed per generator manufacturing unit equals three times the generator's volume.

Similar transportation methods are used to carry raw materials, but no warehousing is necessary.

The cost of transportation by truck from the raw material source to the manufacturer is $3.5 per volume of a generator. Transportation costs from manufacturers to warehouses are $4.0 per generator by truck, $3.0 per generator by rail line, and $1.50 per generator by cargo ships. Transportation costs from warehouses to end users or customers are $2.0 per generator by truck, $2.0 per generator by rail line, and $1.25 per generator volume by cargo ships. There are no waterways for the transportation of cargo ships.

 

Table 1: Total units

Table 2: Total items

Table 3: Total cost

Vehicles and goods lifting machines are very important for a logistic company. The vehicles help the workers by lifting heavy goods which save both, time and energy. So having a wide and good range of vehicles which can lift weight and move around quickly are very important. The complex has vehicle dumpers for quickly accepting rapeseed, corn, and barley and transferring it to the warehouse in piecemeal containers (Sgarbossaet al. 2020). The tools which are used to lift heavy-weight goods like cranes should be used to save time. These machines make the work simpler with the technology. Along with all this, companies should also focus on increasing the speed of their delivery. Businesses must concentrate on delivery times and costs if they want to remain competitive in the market. If a product's manufacturing costs (cost of production) are the same, logistics providers might offer their clients distinct value propositions. The majority of clients will choose a business that offers prompt delivery at a reasonable cost.

By maintaining open lines of inquiry with suppliers and keeping them informed about logistics strategies, physical storage can be decreased. Cooperating with suppliers and providing them with information on the demand pattern will help the company in clearing the deadstock, which is more of a headache for any company. Designing the routing is very crucial so that the goods are maintained in transportation for the majority of the time to lower the cost of the inventory. This also saves time for the company and the supply team.

The average time is 5 minutes to land an aero plane
Therefore, variability of arrival= 5/60+4.3= 4.38
The average rate to arrive in a Poisson fashion= 6/ 60=0.1
The field waiting time= 5minutes
The maximum time spent =5* 0.1+4.38=4.88 minutes to arrive
The pilot will take 4.88 minutes to spend circling in the field.

Storing places like warehouses and storehouses are very important for the transport company. Some companies also deploy a large number of customised trucks which act like a storage house. These trucks are very useful as they give dual advantages to the user. Depending on their design, some trucks can be operated by the operators. When it comes to storage houses, the company needs to have multiple storage houses. It provides the opportunity to store many goods at a time. So, the company should use big storage houses along with these trucks which enables the operators to store goods within them. Generally, companies keep storing houses which helps the manufacturing unit in producing large quantities of goods (Min et al. 2019). The same strategy is used by logistic companies. They keep a large warehouse and store their goods which are to be delivered to the customers. The companies should also modernise their equipment and tools which are time-saving and way cheaper. Substituting information is essential and necessitates taking important actions to build the logistics network.

Finding the appropriate places to locate is the first step. Once every location has been identified, then the company needs to forward and reverse supply chains to determine which locations are most advantageous for efficient and affordable logistics. The company needs to develop an export-import strategy as the next phase is very important. Determining the quantity of the goods being exported and imported, will help the company in getting the required number of freights, and then the managers can decide where to locate the inventory for tactical advantage. The next process includes choosing the warehouse location. By determining how many warehouses are required, and their distance from markets, the company can construct the warehouses in a cost- and time-effective manner throughout the world. The fourth step is to choose a combination of carriers and modes of transportation to efficiently supply the deliveries. The correct amount of partners must be chosen by the company to build a successful logistics network.

Determining the operations' useful procedure can be assisted by an efficient analytical approach. This cluster requires a strong customer link to engage or share capacity, resources, facilities, and information with internal and external customers and suppliers. The correct data about the capacity, facility and resources must be provided. To reduce drawn-out waiting and useless delays, more information must be given to the concerned team. The planningand optimization of transportation goods are the main goal of transportation management, which is an essential component of logistics transportation (Perboliet al. 2018).

After analysing the problem, it was found that the organisation lacks in Road transport. During the covid period, the organisation didn't have enough vessels to carry the essentials. These things are still the same. The problems are even bothering the company today. One important aspect of this analysis was cost cutting. Minimising the cost of less important items is an efficient way of increasing profit. The company should start implementing things which will help in cost reduction. The company should also improve its transport selection. Selecting a good mode of transport for delivering goods is very important for a logistic company. Remote areas that previously had little impact on international supply chain can now make goods and distribute them to markets at competitive prices (Garcia et al. 2018). The transporter can complete the shipping procedure at a lesser cost because of effective mode selection. The cost is only one factor in determining the most cost-effective mode of transportation; other factors include effectiveness and accessibility for both the producer and the consumer. The best balance between cost and speed can be achieved by using an efficient mode of transportation. Costs associated with logistics are influenced by several factors. Some of those elements, including the presence of transportation infrastructures and the accessibility of logistics providers, are controllable, while others are not. These elements evolve as well. For example, customer demands typically shift over time. By regularly evaluating these parameters, one may save logistics costs and enhance customer service.

Question 4

The port also has the facility of an airport. Several planes can be used by the company to transport goods through the air. This will increase the profit of the company and the stakeholders. However, before taking off or landing the pilots and the crew members need to take clearance from the ground staff. Once granted the all-clear to land, an aeroplane takes an average of 5 minutes to touch down at Cairns Airport in Queensland, Australia. A pilot needs to spend an average of minutes circling the field. The maximum time spent circling the field waiting can be reduced by clearing the runway as soon as possible. In this way, the ground staff will take comparatively less time to give clearance to the pilot.The arrival process is considered to be distributed with Poisson, with an average arrival rate of 1.2 passengers per minute at the immigration counters (Di Mascioet al. 2020).

The average time is 5 minutes to land an aero plane

Therefore, variability of arrival= Total time /hour+ time of dispatch= 5/60+4.3= 4.38

The average rate to arrive in a Poisson fashion=Time / hour 6/ 60=0.1

The field waiting time= 5minutes

The maximum time spent = field wait time* 0.1+ 4.38=5* 0.1+4.38=4.88 minutes to arrive

The pilot will take 4.88 minutes to spend circling in the field.

The transportation method is typically used for time-sensitive, perishable, or expensive goods whose final consumers are located far away because it is more expensive than other routes of transportation. Globalization has several primary contributors, one of which is air travel. With a speed that no other form of transportation can match, it allows for the international mobility of people and things. However, above the airspace of certain nations, commercial airliners and air freight are typically restricted to designated fly zones. The hub-and-spoke supply chain system is currently used by the majority of large carriers. This means that large businesses run a primary hub in the middle of their operational area, connecting to more compact regional hubs placed in the same general vicinity but strategically far from it. There must be 8 conveyor belts in order to restrict the baggage claim time to 12 minutes.

Question 5

There are 12 docks and 7 unloading workers in the Victoria port of Geelong. Ships are diverted to 30 km down the coast at the Docklands port region near Melbourne city when all the berths are occupied. According to a Poisson process, oil tankers from the Middle Eastern oilfields arrive on average every 65 minutes. An average tanker unloading crew needs six hours to complete the task, with the time following an exponential distribution. So, from this information, we can conclude that there are not many ships present at the port. The following are the answers to the questions which were asked:

a) The overflow facility 30 kilometres down

Therefore, arrival of shipping = No of ships * 0.30+time=30*0.30+60=69
The tankers required=No of tankers/time= 69/3= 23

On the average 23 tankers are at the port.

b) The arrival of tankers required= 65
The tankers spend at port=65/23 total /number of tanker=2.82 minutes
On the average 2.82 minutes tanker spend at the port.

c) The average arrival rate at the overflow facility= tanker spend at port/time= 2.82*60/65=2.6

d) The probability rate=time of tanker/nu of ships+100=2.82*2.6+65/100=7%

From the above information, we can conclude that the port is big however sometimes the incoming ships are sent to another port near the dockland. The crew is less in numbers but they are very efficient. As they tend to unload a tanker in 6 hours, so on average only six tankers can be present at the port.

This assumes that all passengers have reviewed their bags for claims and that the delivery ratio of passengers at the security checkpoint is equal to the quality-of-service rate at the mass migration counters (Pereraet al. 2018). The port is big enough and it seems the quiet is always filled with tankers, however, there is a probability of 10% that there won't be any tankers on the port. The crew members take approximately six hours to unload a tanker and hence only six tankers can be present at the port at a time on average. 

Reference List

Read More

2128IBA The Management of Business Processes Assignment Sample

TASK RATIONALE.

This assessment item focusses our attention on business processes, value chains, strategic alignment and ethics. It covers materials from Week 8 to Week 12. For the successful development of this assignment, you need to combine theoretical concepts about particular topics and, apply them to the specific situation presented. Use your own words to build your explanations and, academic references to strengthen your arguments.

TASK DESCRIPTION.

Part 1. In this question, you will draw a Value Chain (VC) of a University focussing on the delivery of Online teaching (students attend lectures, tutorials, labs practice and library services solely Online). Hint, see readings of Topics 9 and 10.

 


b) Pinpoint two “strategically relevant activities” and explain why they are strategically relevant.

c) Pinpoint two ‘internal linkages’ and explain why they are strategic.

Part 2 (15 marks). This question is about ethics and business processes. Based on the ‘Online Value Chain’ of the University, you developed in Part 1, address the following points:

(i) Point out one activity in the University ‘Online Value Chain’ (see part 1) that is (or that it could be potentially) related to ethical issues. Explain how or why.

(ii) What type(s) of management ethics can be associated to this activity? Elaborate an argument to support your answer.

Use two academic references (minimum) to sustain your arguments.

Part 3 (15 marks). This is a practical case study about offshoring. This question is independent from previous questions.

You are the new Global supply chain manager at Jabil (one of the major electronic contract manufacturers, see jabil.com). Because of high demand for modems in Europe, you need to decide the location of the next 4500 people factory of integrated circuits for drones. The last two short listed locations are Turkey and Mexico. Assuming labour costs (wages) at the exact same level at both countries, define what the best location for the new factory is.

Your Task: Elaborate a short report (800 words aprox.) to analyse the situation, justify your choice (explain why) and, relate your answer to the support of SDG goals. Consider three relevant non-economic factors to base your selection and two SDG goals. To this end, you must base your arguments on (i) the information provided in the World Competitiveness Report 2019 (see link below) and (ii) research three academic articles (minimum)

Format

• Word format (do not use pdf).
• 2000 (+/- 10%) words max. (all included).
• 12pt Times New Roman, Single space.
• Neither executive summary nor table of contents are required but do include a title page indicating title of your report, full name, Snumber, number of words.
• File format: A2_surname_Snumber example: A2_Guzman_s99999

Solution

Introduction

International business development has been significant for the current business organisations as it helps in increasing better brand popularity and organisational reputation. Understanding value chain and supply chain management are important for setting up manufacturing units and business set up in foreign countries. This study focuses on providing a wide understanding of different factors based on which a proper understanding can be developed about the significance of value chain and supply chain methods as well as business ethics. Through understanding about the details of value chain, supply chain as well as the case of Jabil’s international manufacturing setup can be elaborated.

Task 1

Porter's value chain framework for Griffith University

Table1. Mapping a Value Chain
(Source: Self-created by author)

Firm Infrastructure: Griffith University has over the years enhanced its online working systems and in 2019 has been recognised among sixth highest student’s satisfaction institutions. The organisation has increased its teaching strength to provide education to among 50,000 students in different subjects including a full suite from undergraduate to postgraduate (topuniversities.com. 2022).

Human Resource management

As per Collier & Evans, (2012) value chain points out the flow of services, product or information over which financial transactions happen between suppliers and buyers. The University has increased its strength in HRM by acquiring skilled professors and administration staff to manage the rise in the number of students. Online teaching has observed a big leap after the pandemic when most of the private and non private organisations around the world are forced to close their buildings or any gatherings. The online cycle in the higher teaching sector is primarily focussed to impart professional skills among the students through which they can find a placement (Bruin, 2021). However, the whole concept revolves around ways through which the learning process will take place. This means the administration part in which HRM using modern technology will provide students to procure best market options and in return it will help the institutions to restore their academic value and strengthen online business for best assignment help

Technology development by Griffith University

Technology has a crucial part in this segment and the whole concept of learning depends on the way through which it is imparted. Griffith University has high student satisfaction ratings and is gradually working on the technology development and marketing part where the students from all around the globe can enrol for online education. The organisation has strong infrastructure and management resources which enables it to provide enhanced learning experience. However, online teaching is not adopted purposely by many institutions which impacted on the sudden introduction during the pandemic. The institution is working on parallel programs through which students enrolled can input depth knowledge of the subject as well complete University curriculums in time. The university has collaborated with global giants and placement agencies to ensure proper placement for students and which will create value to the institution.

Procurement

The procurement process both in terms of inbound and outbound activities has increased. The initial task of management to strengthen teaching range has helped to introduce more subjects in the suite of undergraduate and postgraduate. More range in subjects further provided interest among students from different fields to enrol into their relative subject of interest. The management strategy to onboard bigger companies and placement agencies has further strengthened the procurement process (Laryea, 2019).

Strategically relevant activities

Human resource management and procurement process are two strategically relevant activities. Strong HRM policies include procurement of resources and materials in their management activities. For instance the task of management in the Griffith University is not only to align and administer teachers and staff associated, while the management even looks at the marketing & sales, placement positioning, administering the technological enhancements, as well reverting to issues faced by students and teaching staff members. In case with online teaching the value proposition for students is higher. Longevity in the business will sustain only when the institution has the reputation to satisfy students by providing better education as well as imparting abilities to accelerate in the growing market competition. The management in Griffith University has to develop strong procurement strategies, which will fulfil the requirements of technical skilled staff and enable enough to build strong market value by enhancing online learning experience along with better placement opportunities.

Internal Linkages

HRM is internally linked to services and development in the management resources will allow enhancing a better working environment. Skilled and effective management will listen to students' queries and complaints in a shorter time period with impactful action on the problem to resolve it. Strong HRM will work on areas through which better service is gained as the management of Griffith University understands growing market competition in the online teaching industry. Their target is to ensure student satisfaction is met. During the pandemic the management has introduced remote tasks where groups are organised to perform tasks remotely. The management makes sure that students' focus from study does not get diluted.

Technology is internally linked to procurement and services. Use of advanced tools has helped in the marketing where innovative approaches by the marketing teams are used to increase online student’s admission during the pandemic period. The organisation has infused modern tools which allow them to take interactive sessions as well discuss issues in real time with their professors. Use of modern tools has provided virtual imagery of working through which encapsulates belief among students to compete in the new business environment.

Task 2

Ethical issues present and its relation with all parties

Moral use of data and resources has become a bigger challenge especially where the online working system is not the mainframe. Educational Institutions mostly are forced to online during pandemic for which they are not technically able. In a hurry many institutions have used third party applications where the risk of data misuse increases. Data if not properly handled will question the reliability of management practises. Further trust of students will break, bringing the reputation down.

Reducing the accessibility of tools with increase in security patch system

The management can reduce the accessibility to University sites and official websites with increase in security patch by proving security codes to only members. Lack of security patches in the system has further risk for Institution to ensure their secure documents and question papers that they set for students. Apart from various positive impacts that online teaching has created during the pandemic and before that, there are some adversaries present which have put questions on calculating student’s ability to perform in the real platform. As most Universities are forced to take online remote examination systems, the question of data authenticity became a big problem. It is quite obvious that offline education has a serious impact on student development both in terms of behaviour and technical abilities.

Part 3

Justification of choice and its relation with SDG goals

Mexico can be a better place compared to that of Turkey as the SDG index Rank of Mexico is higher than that of Turkey. Based on this factor Jabil can develop its overall business process of setting up a manufacturing unit of 45000 people factory as this has been one of the most effective factors through which a better understanding in business development can be ensured. The SDG index of Mexico is 80 and Turkey is 70 which prove the seriousness and the intention of Mexico to develop its business process and organisational infrastructure in the country (Dashboards.sdgindex.org, 2022). This has been one of the main reasons as it proves the better infrastructure development of the country and marks the seriousness of the government in improving the social development rate of the country by improving its overall SDG goals. In case of both the countries it has been identified that both the countries have the same SDG. However, the rate of fulfilling the SDG is higher in Mexico than that of Turkey. On the other hand, another important factor that sets Mexico at a higher context is based on the global competitive positions that are another important factor providing about the future business development of Jabil in its foreign business market.

The Global Competitive Rank of Mexico is 48 whereas Turkey is in 61st position which decreases the chances of selection of Turkey as it is clear from the rankings about the slow pace development of the country which is one of the pertinent reasons. Based on this perspective Jabil can select Mexico other than Turkey. Lastly, but most importantly the main reason based on which Turkey cannot be selected by Jabil is due to its high rate of labour charge. The minimum rate of labour charge in Turkey is 4250 Turkish Liras ($272) as the Government of Turkey has increased 50% of its minimum labour wage rate (Business-standard.com, 2021). On the contrary, the minimum labour wage in Mexico is about $172.87 MXP ($8.06USD) (Natlawreview.com, 2022). Depending upon the understanding the company needs to spend less wage for the employees if the manufacturing set up is settled in Mexico. This has been a significant reason based on which Mexico can be chosen as the selected country for the setting up of the manufacturing process of Jabil.
Three non-economic factors based on the selection of two SDG goals

 

Table 2: Comparison of three non-economic factors for selection of Mexico
(Source: Created by Author)

Depending upon the overall analysis of the three non-economic factors it has been identified that Mexico is better compared to the position of Turkey as the Global labour market rate of Mexico is 96 compared to the 109th position of Turkey in the global ranking (Weforum.org, 2021). This factor proves that the labour market is better in Mexico as the rate is less establishing the fact about the strong labour market hold in Mexico based on which increased shortage of labour can be a rare factor promoting better manufacturing and business growth. On the contrary, based on skill set Mexico is again in a better condition as the rate of expenditure of skilled labour wages in Mexico is more than that of Turkey. Mexico’s rate is 17200 MXN per month which is more compared to 2830 TRY per month of Turkey (Tradingeconomics.com, 2021). This proves that the condition of Mexico is better than that of Turkey as more skilled employees are pre44 sent in Mexico compared to that of Turkey.

Lastly, the health rate of health development is also slightly higher in Mexico compared to Turkey as Mexico’s health development rate is 54.7 compared to the low rate of 53.9 of the Turkish people (Ghsindex.org, 2022). Health development rate is an important factor for companies as the aspect of labour health can be considered as an important factor. Based on this condition, international business organisations consider labour health conditions as an important factor prior to setting up a business particular manufacturing set up unit in a particular country. This has been one of the important factors based on which Jabil can engage in choosing Mexico as it will be getting better development in production results in Mexico compared to that of turkey. In every aspect it has been identified that Mexico is far better than Turkey based on both economic as well as non-economic factors. Depending upon all these aspects, it is important for Jabil to choose Mexico for setting up its manufacturing unit of integrated circuits for drones.

Conclusion

Thus, it can be concluded that there are various important factors based on which international business organisations set up their manufacturing units in different locations. It is important for business organisations to understand the significance of supply chain management through value chain analysis to gather a productive idea related to the different organisational aspects important for setting a business process in a foreign market. Proper understanding of different ethical factors also have been identified which increases the core understanding of business internationalisation. The case of Jabil also has been identified which helps in providing a proper understanding about different economic as well as non-economic factors based on which an organisation can set up its business process in a foreign location.

Reference List

Read More

Design Management Audit and Employee Engagement Plan Assignment Sample

Assignment Learning Objectives:

At the end of this assignment students will be able to:

• Explain the drives that motivate and engage employees and how to fulfill them.

• Design and apply strategies for motivating employees in the Workplace

• Explain how to build culture of Employee engagement and participation

Case Scenario:

• Sarah is the vice president of HR at ‘AccountingFast’, a midsize financial services company.

• AccountingFast is facing a major problem with employees’ satisfaction, engagement, and retention.

• Sarah has hired you as their HR consultant to urgently tackle this major problem.

• Sarah wants you as their HR consultant to create and roll out a management audit that will help her uncover the reasons behind the high employees’ turnover and then based on the management audit results, help her put new employees’ engagement programs and policies in place.

o Here is the information Sarah gave you in your first meeting with her:

 “Nearly 100 employees had given the notice to leave the company in recent weeks.”

 “Our turnover rate is up to 35%.”

 “We are growing — in revenue, profitability, and reputation, I just don’t understand why people are leaving us fast,”.

 “I need to present these numbers to Mr. Richard Leo (CEO) at the end of the week, and I can’t do that without a theory on what’s happening and a solution to propose. That’s why I called this meeting,”.
Assignment Instructions:

• You can make any appropriate assumptions about the management audit results that will help you complete your research and assignment.

• This assignment is to be done individually. Carefully read the scenario below and write a 1,800-2,000 -word research report In MS Word, double spaced, 12-point font, APA standards.

• Your case analysis must include a minimum of 10 external academic resources.

Assignment Questions:

Part A:

Your task is to carefully design the management audit questionnaire in the light of theories/concepts/Canadian

Laws discussed in the class.

• Explain the different sections and elements in your management audit design.

• Explain the purpose and relevance of the different management audit questions and how the questions will help you to know the causes of employee turnover/dissatisfaction.

• Explain the reasoning behind your design using academic research, the course concepts, case information, and critical thinking.

• Helpful tips:

1. You can include the actual management audit questions in the appendix section, so you don’t run out of word count.

2. Create your own questions from scratch. Copied questions from the internet and other sources will be marked down.

PART B:

• Based on areas of improvement identified through the management audit in part A, design and create an employee engagement/satisfaction plan for Accounting Fast that will help them reduce employee turnover and increase employee engagement.

• Explain the reasoning behind your suggested plan using academic research, the course concepts, case facts and critical thinking.

Solution

Introduction

Accounting fast is a midsize company that provides financial services to other companies. It is growing substantially at a high speed; it is generating great revenues also. It is a profitable company. It is planning to open more branches across the country. In spite of these positive sides of the company, it has a very high employee turnover rate of over 35 percent. To mitigate these problems, Sarah, who is the vice president of the human resource department at the company hires a HR consultant. HR consultants will create a management audit. The results of this audit will help Sarah to present a report to the C.E.O of the company incorporating reasons for the issues and the solutions to resolve those.

Part A

Management audit

Accounting fast needs to conduct internal as well as external audits for generating greater insight into the organisation. As stated in the Canadian accounting standards 200,210, 220 this audits are very time-consuming and cumbersome but are very helpful in understanding the internal problems of the organisation. The company can identify the critical problems which are restricting the company from long-standing sustainability (Tsourvakas & Yfantidou, 2018). There are mainly 6 segments in a management audit. These are as follows:

Planning

To create a proper management audit pan, internal auditors at first need to set the objectives. After that, a timeframe is to be established (Lennox, Wang, & Wu, 2018). Then, the auditor needs to schedule meetings with the human resource management of the organisation. Then, they ask for documentation about the problems for assignment help.

Document review

According to the CAS 265 for communicating the deficiencies in the internal control system of those incharge of the governance system, the auditors thoroughly examine all the review policies, established controls and procedures (Wahyuandari, 2019). The main thing factors here are the auditors see how the employees get rewarded after doing an excellent job. Then they see on what basis the organisation gives promotions to its employees.

Field work

In this stage, the management auditors do all the physical jobs. They come to the organisation to examine whether the policies are aligned with the documentation or not. Then, they interact with the employees directly and listen to the problems they are facing. They organised interviews with employees to know why the employees are not satisfied with the company and why they are not engaging with the company (Sohal et al., 2020). They also check whether all the organisations are able to maintain a friendly environment at the workplace or not.

Follow up

The CAS 450 involves dealing with any msitatstemte identified in the audit work and the CAS 402 relates to Audit consideration resting to an Entity using a service. Thus the management auditor asks for all the missing documents he/she needs to finish this sudit. The auditors should have the right to access the review reports (Milliman, Gatling and Kim, 2018). If there is a contradiction between the employee's answer and management’s justification, then the auditor requests further clarification on that. He/she needs to eliminate all the confusion before submitting the audit to the higher authority of the company.

Reporting

In this stage, the auditor reviews the information presented to him/her both from the management side and employee side properly (Iriyadi, 2019). According to the CAS 500 audit evidence needs to be submitted and a kind of draft report will be issued by the auditor. It includes the final outcome of the audit. This report critically evaluates the strength of the organisation's programs. It also incorporates finded weaknesses of the company. Finally, the report gives recommendations to correct the action plans.

Tracking the issue

The company does not implement any strategy for employee retention while hiring a group of employees. Additionally, as the company's main focus is on earning more money and enhancing brand image, it often does not care about the relationships between its employees. Absence of onboarding culture (Píchová & Raušer, 2018). Employees mainly leave acoountingfast because they do not think that staying in the company will advance his/her career. The work culture of the organization is not diverse, it does not value the opinions of the junior employees. Moreover, the organization is not mobile at all. Without having any chances for growth, employees are leaving the company in search of expanding their knowledge, expertise, and strength.

Another important issue is the salary. The employees are paid less than the employees working in other companies doing the same job The employees are also exhausted with a bust time schedule. So, improper work-life balance is a major issue.

Moreover, the company does not have a compassionate leadership practice. Which is mainly contributing to this high employee turnover rate. Discrimination in the company between male and female employees (Píchová & Raušer, 2018). Often, male employees get preference during promotion or remuneration over women employees. Lack of proper policies which secure the rights of employees including anti-harassment law.

Part B

Identified employee engagement plan

Implementation of Maslow's hierarchy of needs theory
By implementing this theory, an organisation can effectively identify its employees' needs for satisfaction. This suggests the needs of the workers should be met in an orderly manner. For increasing productivity in the team, the organisation needs to assure that all its employees get a hundred percent motivated to do the job more efficiently. A better work environment can be fostered by this. Due to the ongoing pandemic, most of the company’s employees are working remotely from their homes (Tsourvakas and Yfantidou 2018). So, Accoutingfast, should make sure that all its employees are mentally fit. Maslow’s hierarchy can be applied to make strategies for this. Furthermore, the employers should feel that their future is totally safe in the company. Sometimes, a regular salary does not give me the feeling of security. As a result, the company should mitigate this problem by giving a full job contract.

Implementation of Herzberg's two-factor theory

This theory is best suitable for creating a sense of self-satisfaction amongst the employees. The company needs to recognize the employees who have done tremendous hard work for the benefit of the company. Thus, the employee will feel accomplished as well as appreciated. This will also encourage the employees to continue the hard work and reinforce better performance. The authority should present a precise plan on how the employee will progress within their position in the organization. Thus, it will give a boost to employee productivity. To maintain a healthy work environment, there should be zero tolerance for anyone who acts inappropriately or disruptively (Siddiqui, 2019). To improve working conditions, the company should clarify the pay scale, employment contract, and working hours and stick to that. Moreover, an employee's status has to be maintained within the organization. To implement this model. All the factors which are causing dissatisfaction among the employees should be eliminated. Management should check all complaints of the team member whtete they are appropriate or not (Hassan, Zailani & Hasan, 2019). Team members should be allowed for opportunities that assist the team reskill themselves.

Application of Employee Engagement theory

This theory supports and inspires employees. Thus, employee outputs have increased by many folds. The organization needs to give more priority on making its employees loyal to the company. It will result in less absenteeism. The most famous employee engagement theory is “William Kahn”’s engagement theory. It harnesses the organization's members. As a result, employees can express themselves cognitively as well as emotionally in the workplace. It provides the basic roots of team building. The management should be concern about employees, make social ties with employees (Siddiqui, 2019). Thus, employees' motivation can be greatly enhanced. Moreover, the company can increase meaningfulness in its products. When an employee thinks that he/she is contributing to society through the company's products or services, they can work with more dedication. Thus, they will feel the work is reasonable. Employees should be given tasks according to their capacity.

Moreover, AccountingFast should give more work-life balance to its employees. They should provide more vacations, leaves to the employees in which they can spend time with their family and friends. Thus, they will be mentally more relaxed. With a fresh mind, they can think more creatively. This will act as a competitive advantage for the coolant over its competitors. This increases job satisfaction as well as productivity which in turn reduces employee turnover rate.

Based on the Canadian Human Right Act law, the organisation should focus on creating a workplace where every employee feels engaged, fulfilled and happier. The senior, junior and highest paid employee all should be the same enthusiasts for the betterment of the organization (Bušatli? et al. 2018). By applying this theory, accountisgfast will be clear about different segments of the organization and the trend of changing motivation over time. Thus, it will be successful to identify the factors which are demotivating its employees.

Application of Ulrich Model

This model suggests that the organization should implement a unified structure. Thus, clear distinctions will be defined for the management. Thus, the company can analyse whether it has performed as per metrics or not. The organization gains a strong benchmark from this which is very helpful in operating business. This model is very effective in maintaining the management functions seamlessly (Hassan, Zailani, and Hasan, 2019). By properly understanding this model, the organization can balance transactions more efficiently with customer expectations. Moreover, it clearly defines the role of management in the company which leaves no scope for any confusion.

In the past, HR only did the administrative side. But, in today's world a successful organization's HR needs to focus both on the administrative and strategic side. The Ulrich Model effectively helps in this case. It is essential to recognize and constructively appreciate an employee's work. This model makes HR smor competent. As a result, HRs will be able to add more value to a company's business. Management needs to serve talent beyond customer experience and productivity. The model teaches the management how to embrace competence, contribution as well as commitment (Abraham et al. 2021). The model incorporates morphology, capability, ecosystem capability, and alignment within the organization's culture. In accordance with the Employment equity act, embracing a type of mixed culture, it draws a thin line between transactional work and strategic work. Aligning innovative and improved strategies management will be capable of integrating all the strategies. Now in this technologically advanced age, the theory helps the management in creating business strategies. Thus, management can become faithful strategic partners.

Action plan

Conclusion

“Accounting Fast” has many issues to resolve to improve its business operations. The company should focus more on the right hiring strategy. It should look forward to hiring the appropriate employees. Because 100 people had left the company in recent weeks, this has caused a huge backlash for the company. As it had lost a valuable part of their team. The company needs a vast change in its work culture. For this they need to implement different theories and models as mentioned above. These will help the company to reduce employee turnover rate which was the main problem to a great extent. In this way, the organisation can achieve its future business goals without any restriction.

Reference List

Read More

HRM202 Human Resource Planning and Development Assignment Sample

1. Discrimination and contract of employment

Jacinta is a young woman has been working regular casual hours for the last 2 years. She asks her employer to convert her to a permanent part time employee. Her employee tells her that she does not qualify for this as her hours are not regular. The following week, Mary’s hours are reduced, and in subsequent weeks the hours are increased back to the previous level, but with different start times.

- In your opinion for assignment help has the employer acted unlawfully in this situation? Why? What action could the Jacinta take?

2. Wages Theft

- Bepo was employed by Tong’s Chinese in one of its food court shop in a major shopping centre. She worked ever Saturday from 9am until 9pm, with a two hour break in the middle of the day, and Monday and Tuesday from 10am-2pm, without a break. She received a flat rate of pay of $ 12 per hour and was not provided any casual evening or weekend loading, and was told that she did not qualify for holidays. Talking with friends, she realised that she was being paid much less than they were.

- In your opinion has the employer acted unlawfully in this situation? Why? What action should Bepo take?

Solution

1) The action taken by the employer was unlawful because the employee has been discriminated from the other employees. Under the Fair Act of Australia, discrimination against any employee is regarded as an unlawful act by an employer. This includes part-time and casual employees, full time, trainees or learners, and any individuals for fixed periods or tasks. Before reporting this case to the concerned authority, she should let her employer get an idea that she is taking the matter very seriously. She should make a report stating all the time when she was discriminate. The report should contain the correct date, approx. time, people involved, location, witnesses, and details of any improper conduct or any speech and if she does not get any positive response from the employer then Jacinta should report the discrimination immediately to the company&#39;s management because when an employer is charged by a discrimination complaint, they are obligated by the law for investigating it promptly. Getting the law involved in the case, and having the state contact your employer, will help her in getting someone’s attention quicker. Discrimination can be done on various grounds like religion, race, caste, place of birth and sex. To remove discrimination in the workplace one should always focus on their strength because focusing on their core values will motivate them to succeed and will help in removing all those negative thoughts and discrimination from the mind. Overcoming all these hardships will also help in facing future challenges. A person experiencing any kind of discrimination should immediately report the issue to their direct supervisor, manager or director. In order to gain the trust and credibility of one&#39;s company, workplace discrimination issues should be solved in a timely manner. A company should educate and train their workers about what constitutes discrimination.

2) The employer has acted unlawfully because the deduction was unreasonable and also the notice of termination was not given to the employee. The employer cannot deduct the money if the employee has not agreed to the writings and if it is not allowed by the law i.e., the Fair Work Commission, or by any sort of court order. A deduction is said to be reasonable only if an employee is not providing goods or services to an employer as a part of their business but if the employer is paying less even if the work was done all according to the principles stated, then the deduction is regarded as an unreasonable deduction, which comes under an unlawful act. The Fair Work Act, of 2009 says that the employer can only deduct up to one week&#39;s wages from an employee&#39;s pay under the following circumstances which include: if the employee is over 18 years of age if the deduction seems appropriate and if the employee has not provided the right amount of notice under their award. Bepo should immediately take action against it. Under the Fair Work Act, 2009, it has been mentioned that the employees should start off the court proceedings if they want to recover the underpaid wages within the six years from the date of when the underpayment started so either she should go to the court or either she can try to resolve the disputes by directly communicating to his employer under whom she is working. According to the FW Act, it has been mentioned that an employer cannot take any adverse action against the employee if they have been charged for underpayment because it is a legal right provided to an employee. One more option that she could opt for even if the employer does not increase the payment is to quit that job.

Read More

BUS3002 Business School Industrial Experience Assignment Sample

Assessment Details

A new Australian Tablet Store recently opened to sell iPads and other tablet computing devices. It purchases its tablets directly from the manufacturers. To order tablets, Australian Tablet Store employee submits a purchase order to the manufacturer electronically. Each purchase order could stipulate several different models of tablets from one manufacturer. The manufacturers typically deliver the tablets to the store within two weeks after they receive the purchase order. The Australian Tablet Store pays for each shipment within 30 days after receipt. If there are multiple orders to the same manufacturer, the Tablet Store occasionally combines payments, issuing one check for multiple receipts. All the Australian Tablet Store cheques are drawn on one bank account.

On the sale side, Australian Tablet Store sells only iPads directly to the customer from its only outlet located in Melbourne CBD. Each sale needs approximately take 30 minutes and customers often buy more than one product. Each sale is identified with a unique invoice number. Majority customers pay on the counter; however, some customers also ask for credit sale, payable in full within 30 days. Customers sometimes combine payments for multiple sales.

You are required to prepare the assignment based on week-2 to week 8 lecture materials and class notes and demonstrate understanding towards the following questions.

1. How would the Australian Tablet store use business intelligence to monitor its business? What source of data needs to be gathered and how would they be analysed? (800 words)

Important Instructions:

- This is a group assignment and due at the end of week 9. Only one group member is required to upload the assignment files to avoid plagiarism issue.

- There will be maximum three members in one group. All group members’ names and IDs must be mentioned on the cover page.

- Students are also required to include proper introduction, conclusion and references in the assignment.

- Final submission must be in one single word document.

Solution

Monitoring business using BI and source of data

Business Intelligence (BI):

BI is a set of techniques, structures, and software which converts unstructured data into usable data. It combines services and analytics for transforming data into actionable insights and data. BI impacts a firm's operations, tactical and functional business decisions. Instead of depending on speculation and intuition, BI fosters fact-based judgement relying on previous data for best assignment help.

Use of Business Intelligence by the Australian Tablet Stores:

The use of business intelligence has been discussed below:

1. BI allows businesses to quickly extract actionable insights from current and historical data to make strategic decisions. This is made possible by corporate analytics systems, which analyse large amounts of data from various sources and present insights in aesthetically appealing, easy-to-understand formats (Zamba, et al., 2018).

2. Business intelligence can help Australian tablet stores make smarter choices by providing current and historical data within the context of their operation. Business intelligence can help Australian tablet stores deliver efficiency and comparative standards, enabling businesses to operate more properly and effectively. Analysts in Australian tablet stores will recognize market dynamics faster, allowing them to increase revenue and sales (Salur and Kattar, 2021). If utilized appropriately, the proper statistics can help with anything from regulation to hiring strategies. Following are some instances of how business intelligence could help
Australian tablet shops make better, data-driven choices:

- Determine ways to boost profits.
- Examine the behaviour of their customers.
- Compare information with that of competitors.
- Keep track of the progress.
- Streamline business processes.
- Determine the chances of success.
- Recognize market trends.
- Identify difficulties or problems.

The practice of utilizing information to make choices can encompass any aspect of a business. Applying BI principles to convert unstructured data into valuable information can help a group make better decisions (Lennerholt, van Laere and Söderström, 2018). It can assist businesses in reacting fast to shifts in the economy, client preference, and supply chain management.

Data Sources and Data Analysing:

The data that businesses use for business intelligence originates from various sources (Kalna and Belangour, 2019). Internal and external data streams have been the most prevalent kinds of data sources for BI, and they are as follows:

1. Internal Information: Businesses can harvest data from internal resources in several ways. Here are a few descriptions:

2. POS data and transaction information: A corporation's accounting and operational platforms are among the most important sources of information. Australian tablet stores can utilise this data to collect historical and current data regarding their internal commercial activities, and also data about their customers' purchasing patterns (Kalna and Belangour, 2019). These data can provide a business with useful information, such as cost-cutting and budgeting strategies, and crucial trends relevant to their customers' buying patterns and interests.

3. System for managing customer relationships: In addition to purchasing and browsing information, companies can harvest data inside their CRM software. Customer associations, locations, and other geographical or territorial factors may aid in determining the location of clients (Isazad Mashinchi, Ojo and Sullivan, 2019). Whenever these CRM data are combined with transactional data, they become far more powerful.

4. Internal reports: Internal records within a firm are growing increasingly more important than before, particularly in cloud technology. Internally digitised documents can be a vital supply of data, particularly related to the corporation's operations, standards, and processes (Quamar, et al., 2020). As per an infographic by Kapow Software, emails, XML, documents, Pdfs, and a range of many other corporate records can all be harvested for big data.

External Data:

Some of the external Data sources for the BI which Australian tablet stores can use are as follows:

1. Use of social media: Outside information sources such as Twitter and Facebook are among the highest dependable data. These sites have become immensely famous (Quamar, et al. 2020). Australian tablet stores can use social media accounts to better understand their current and prospective customers.

2. Data from the government: Although social media data is unquestionably useful, it is not the only outside data source that firms should consider (Isazad Mashinchi, Ojo and Sullivan, 2019). The national government also provides important data resources to help modern enterprises better comprehend the market.

3. Google and Google Trends: Google also has rendered a few key information sources accessible to the public. Among the most prominent search services on the globe, Google has a wealth of information regarding search terms, patterns, and other digital activities (Salur and Kattar, 2021). Google Trends is among the finest sites available. It offers statistical information on search terms for almost any keyword, and all these datasets stretch back to almost the beginning of the web.

Companies like Australian Tablet Stores can harvest information in many methods, both internally and externally. Transactional data & POS data are two very important sources of data discussed in the paper. Internal data have never been more valuable, especially in the age of cloud computing. Businesses can utilise this data to better understand their activities and purchase patterns. External data such as social media, the internet, and data provided by the government also play an important role that has been precisely explained in the discussion.

References:

 

Read More

BST714 Strategic and Operational Decision Making Assignment Sample

Purpose

To give you an opportunity to obtain first-hand experience in

1. Employing strategic management concepts and analytical tools.

2. Appraising strategic choices and develop and evaluate viable strategic options.

3. Demonstrating a comprehensive understanding of the salient issues in the implementation of strategic change in the internal and external organisational environments.

4. Identifying and evaluating appropriate strategies to manage stakeholder demands.

5. Developing project plans to enable strategy implementation.

If you take this assignment seriously, you will teach yourself many basic principles on strategy, which will stand you in good stead in many subjects, and in many walks of life.

Tasks

1. Review the marketplace / environment characteristics for the sector allocated by using any public domain information available and relevant academic literature search. You will need to select a particular case company to undertake tasks 2-

4. Highlight secondary data sources that you will exploit and explain their strengths and weaknesses.

2. Highlight and evaluate the corporate strategy of the organisation selected. How does the corporate strategy compare to that employed by Tails.com?

3. Providing empirical evidence and analysis as appropriate, determine the order winners and market qualifying criteria and classify the supporting operational process using the Hoekstra, and Romme (Buy-to-Order, Make-to-Order, Assemble-to-Order, Make-to-Stock, or Ship-to-Stock) process categorisation system. How do the order winning / market qualifying criteria and the supporting operational process compared to those of Tails.com?

4. Develop a change management plan to align the case company’s operations strategy to that of Tails.com, with due consideration of the necessary change levers. Is your change plan targeting short-, medium- or long-term change? What are the resource implications of your plan?

Solution

Introduction

Pet are becoming an essential part of the lifestyle. In the UK, around 51% of the household owe pets, out of which dog is the most popular pet type. In the year 2021, around 26% of the residents in the UK have dogs as their pets; on the other hand, 24% of the household have cats. This has led to the boost of the pet sector within the country (pdsa.org.uk/get, 2021). The care stores, as well as the pet post business, are increasing rapidly. For Assignment Help Products like dry pet foods, flea and tick medicines as well as pet grooming suppliers are in huge demand among pet owners. In the process of excelling in the pets sector, it is essential for the companies to understand the condition of the market with an effective understanding of the macro environment of the country to understand the pet sector in a coherent manner. The execution of the corporate strategic development by the comparison of the strategies with the rival companies are effective manner. The study is focused on the business of Stella & Chewy's, which is operatives in an effective manner in the pet sector. The measures are taken to outline the current strategies used by the business and thus suggest alternative approaches for business improvement by understanding the current strategic measures adopted by the company to compete with Tails.com in a vivid manner.

Criterion 1 Context

Marketplace Review


Figure 1: Top ten pets
(Source: Pfma.org.uk, 2021)

As the above picture reflects, the craze for pets is increasing among the customers. Pet is becoming a companion for the people, and they are willing to take good care of their pets. In the UK, around 12 million dogs and 12 million cats are used as pets. On the other hand, the pets like birds and horses are also in high demand (Pfma.org.uk, 2020). The Pet industry in the UK is very promising as it includes Food products, grooming and toys as well as medicines businesses. Taking into consideration of pet’s food products; a diverse growth is evident in the market. As per the national pet owners survey, in the year 2020, around $103.6 billion was spent on pets, and the estimated expenditure on pets in the year 2021 is estimated to be $109.6 billion (Pdsa.org.uk/get, 2021). The organic food business is leading in the pet sector. The companies tend to introduce natural and organic canned food products. The demand for dry food for puppies and pups are also very potential business in the UK market in recent times. The pet industry in the year 2020 has higher sales and has reached $103.6 billion. The achievement is considered to be historic in nature. The sales elevation has been around 6.7% from the sales the sector had in the year 2020 (Supermarketnews.com, 2021). The positive sales and growth of the business in the pet sectors are effective in contributing towards the development of growth measures of the business and thus are acting as the pull factors for the companies to attract investments in the sector and gain positive returns.

In the global market context, the pet care market is considered to grow from $216 billion in the year 2020 to a threat of $232 billion in the year 2021. The pet food market size is also experiencing huge growth. It has been evident that pet food sales have risen from USD 87.08 billion to USD 92.66 billion in the year 2020. It is estimated that the global food market is sure to elevate by 4.6% from the year 2020 to 2027. The estimated returned in the year 2027 is approximately 124.9 billion (Fortunebusinessinsights.com, 2021).

In the views of Morelli et al. (2021), pet owners are wailing to buy healthy food products for their pets, and this tends to provide them with proper nutrition. The business of Stella & Chewy's is also a very promoting business dealing with the nutrient food products for a pet. However, in the process of surveying the market, the evaluation of the macro environment characteristics of the pet sector in UK markets is essential.

Environment Characteristics for the Sector

Political Factors

In the process of understanding the macro-environment factor, the political factor is essential for a business. The stable political condition of the UK is allowing the pet sector to flourishing in an effective manner. The country is supporting the adoption of pets and thus nourishment them with healthy food and food life. The care centre is also supported by the government to protect the animals (Bowley, 2017). The flexible relationship with the potential markets like USA and Australia is allowing the business-like Stella & Chewy's to export their products even in the international markets.

Economic Factors

At the same time, the economy of the country plays a vital role in boosting the pet sector. Pauliuc and Fu (2018) stated that a lot of money is needed for the caring and feeding of the pets; thus, it is essential for the owners to be economically stable to take proper care of the pets. In the case of the UK, the country has a positive GDP rate contributing to the evaluation of the purchasing power of the customers. The increasing percentage of pet owners in the UK are supporting the economical stability that exists in the UK.

Social Factors

Considering the social factors, the people in the UK are increasing care for keeping and caring for pets (Howe and Easterbrook, 2018). The pet industry is present in all sizes in the UK market. The demand for natural food products in the UK market is increasing. The growth of 4.6% is estimated by the year 2027. As per the American Pet Production Association, the sales of pet food products are adding up to US$3.7 billion in the year 2021 than that of 2020 (Fortunebusinessinsights.com, 2021).

Technological Factors

Davies et al. (2019) commented on mentioning that the increased technologies in the UK are promoting the business operating in the UK markets. This is acting as the boom to develop sustainability to the sector and thus allow reduction of waste generation at the same time. The positive growth of the pet sector is evident with the rise in the sales of pet food products. Thus it can be stated that the UK market has the potential to create an effective measure in ensuring success to the business of Stella & Chewy's as the markets in the UK are very competitive for the pet sectors, so the evaluation of the current strategies and its comparison with the competitors is a must.

Criterion 2 Corporate Strategy Development

Stella & Chewy's is one of the leading businesses in the pet sector. The company is using the world's finest ingredients from the supplier’s 100% organic fruits and vegetables are used in the preparation of pet foods. The company is passionate about the absolute best for pets (Stellaandchewys.com, 2021). The execution of the 100% competes, and the balanced diets to the pets thrive. The Fortified with the vitamins and the minerals in the process of ensuring the pets gets the proper nutrients as per the needs of the pets (Stellaandchewys.com, 2021).

Current Strategies

Vision: Stella & Chewy's is determined of providing a high quality of natural pet food with the proper emphasis on nutrition, safety, conveniences as well as palatability (Stellaandchewys.com, 2021).

Objective: Stella & Chewy's is willing to ensure seamless execution of the pet food business and capture a wider market share with the help of strategic making and quality products measures.

• Resource Allocation

The company has used its resources to get fresh and natural marketing from the suppliers to develop the world’s best quality pet foods. The materials are responsible sources, and there are no added hormones or antibiotics (Stellaandchewys.com, 2021). Cost is also invested in marketing the products. The use of blogs is designed to keep the customers aware of the effective strategies used by them in competing in the context of the global market with the expansion of the market share in the UK.

• Organisation Design

Stella & Chewy's is using flat business strategies. This allows the company to take up the fast decision that enables the company to execute excellent service measures and thus build an excellent working strategy in creating fast decision making (Petfoodindustry.com, 2021). The company is able to take a business decision without much confusion as the employees of the different strategies are empowers with decision-making ability in the sport to handle situations and keeping the customers satisfied. This is allowing the business to build locality among the customers, and thus the retention of the customers is possible by the business of Stella & Chewy's.

• Portfolio management

Stella & Chewy’s is in need of effective management. The increase in the employee’s turnovers reflects the outcome that the business is not having n effective team of management (Petfoodindustry.com, 2021). The poor policies and inferiors of healthcare facilities for the workers are making it difficult for the workers to execute their services within the company.

• Strategic tradeoffs

The delivery on its position is essential for Stella & Chewy's. The company has a strong network to deliver the products in different locations. The retail stores are also selling the pet food products of Stella & Chewy's, allowing the product to reach the customers easily.

Comparison with Tails.com

The comparison of Stella & Chewy's and its competitors such as Tails.com executed the fact that Tail.com has an excellent hold on the internal markets whereas Stella & Chewy's is mostly successful in its home country (Petfoodindustry.com, 2021). Tails.com being a UK based brand, has been able to capture huge maker areas in UK market with its excellent business executing strategies (Tails.com, 2021). Stella & Chewy's needs to develop better approaches for the international audiences in the process of gained profitability. At the same time, Stella & Chewy's is having issues in decision making due to the inefficient management that are increasing confusion among the staff.

SWOT analysis

The evaluation of SWOT analysis has highlighted that the business needs to work on the internal management and thus develop strategies for markets expansion with the proper financial allocations. Stella & Chewy's has a huge prospect of growth in the international markets provided that the business is effectively using its strategies measures while entering the new market (Stellaandchewys.com, 2021).

Alignment of the Implication and the Solution

The use of effective leadership with the employees training has the ability to bridge the identified threat. The company needs to identify the proper entry mode that is helpful in executing seamless expansion of the business and thus ensure that the company is able to maximise its sales. Stella & Chewy’s will be able to work on its strategic measures and develop competitive advantages to compete in the UK market. Grover et al. (2018) mentioned that strategic measures are essential in creating a positive attitude among the business. It is thus essential for the management to build its efficiency and create better decision making. The company needs to understand the market demand and, at the same time, create stronger measures in building profit-building measures.

Criterion 3 Operations strategy analysis

Porter generic

• Cost Leadership Strategy

The business of Stella & Chewy's is focusing on the increase in the products. In the opinion of Esmaeel et al. (2018), the increase in the products with the limited cost is helpful for business growth. Stella & Chewy's is the first to adopt the specification in the pet food production, and thus the cost leadership is evident in the case of Stella & Chewy's. The increasing market completion is reducing the cost leadership of Stella & Chewy's that the business needs to work on. The companies like Tail.com are leading the cost leadership, acting as the threats to Stella & Chewy's in the UK market.

• Differentiation Strategies

The company is effective in developing differentiation. The use of the strategies norms is used by the business. The company is able to create specification food products for pets, especially dogs and cats. The food with different speciation and amounts are available. The company aims to use only natural ingredients for food products and therefore tends to execute a wide variety of nutritious pet food products (Stellaandchewys.com, 2021).

• Cost Focus Strategies

The low cost and high quality positing of the product are unique in the case of Stella & Chewy's. It is essential for Stella & Chewy's to low its customers. The demand of pet owners is changing, and Stella & Chewy's needs to make changes in its products as per the market needs and trends (Stellaandchewys.com, 2021). Innovative machines with proper employees training are needed so that the staff can contribute their part in creating customer satisfaction.

• Differentiation Focus Strategy

The use of Innovative technologies such as AI in Stella & Chewy's is needed. Currently, the mechanism used for the food production for the pets are not modernised and thus, the execution of differentiation strategies raw facing issues (Stellaandchewys.com, 2021). Stella & Chewy's needs to work on the inclusion of more variety of food products for the bets and increase their specialisation for the horse and birds products as well.

The use of Porter's generic model has been helpful in creating a better idea about the operation strategies used by Stella & Chewy's. The companies have an effective differentiation in the product, but the re-design of the products as per the current market needs is essential.
Marketing mix

• Product

The products like cage-free chicken raw coated kibble puppy dry dog food, grass-fed beef raw coated kibble, raw coated kibble wild-caught salmon recipe and raw coated kibble cage-free chicken recipe are proposed by Stella & Chewy's (Stellaandchewys.com, 2021). The quality of the products is maintained, and natural raw materials are taken to ensure that the business is able to provide the correct nutrition’s to the pets as per their requirement and thus take the measures to create better customer engagements.

• Price

The use of low price and high-quality products are taken into consideration. The products range from $30 to $ 90 (Stellaandchewys.com, 2021). The price of the product varies on the basis of the ingredients used and the size of the package. The company maintains a low prices strategy so that it can keep the customers satisfied, but the availability of similar products at cheaper rates is acting as a huge market threat to Stella & Chewy's.

• Promotion

Stella & Chewy's is active in creating blogs and making the customers using their webpage (Stellaandchewys.com, 2021). The use of social media marketing is also adopted by the business so that they can develop better customer engagement. This allows the business to create better strategic measures and therefore fetch a wider mass in the limited time period. In the era of digital promotion, Stella & Chewy's is using its social media presence to inform the customers about the new launches and the organic food specification without any preservatives to customers.

• Place

Stella & Chewy's has its headquarter in Oak Creek in the USA, and it has good control over the UK market and is successfully operating with its strategic business executing skills (Stellaandchewys.com, 2021). The network of distribution is used by Stella & Chewy's to reach the leading retail stores and thus make the product easily available to the customers.

Technical skill

In the views of Cooper (2019), the execution of technical skills is essential for the business to build its supremacy in the competitive market condition. In the context of Stella & Chewy's, The Company is not efficient with technical skills. The need for adopting artificial intelligence and developing excellent staffs training is felted. The staffs need to train to develop technical skills and at the same time build confidence in developing strategies measures in creating completive advantages.

Problem-solving ability

Funke et al. (2018) the execution of effective problem-solving skills is considered to be the key to success. Stella & Chewy's is not having a stronger management team. This is affecting the problem-solving ability. The business has, however, adopted the flat business structure, but the inefficient capability of the Stella & Chewy's is restricting the employees to take proper decisions. It is essential to train the employees to develop a stronger attitude in taking on the spot decision and reduce the business instability that arises due to the efficient problem-solving ability.

Criterion 4 Operations Strategy Implementation

Alternative approaches

The discussion in the above sections has pointed out that Stella & Chewy's is failing in maintaining internal integrity among the team. The unstable internal business environment is also the reason for poor decision making. The rival company like Tail.com is very much focused on staffs training, and thus they are able to build a better engagement with the customers fetching profit at the global markets. The dominance of Tail.com in the UK market is increasing the need of adopting alternative approaches. In the case of Stella & Chewy's, the company would be benefited from the product development approaches by staff training.
Change management using Kotter’s eight-step model.

The use of the eight-step model of Kotters is adopted to introduce the alternative business approaches of Stella & Chewy's in a coherent manner.

Step 1: Create Urgency

Stella & Chewy's identified that the competition in the market is increasing. The rival business is using better innovation with the help of their skilled team and is thus able to keep its customers satisfied. The increase in customers turnover and the lack of loyalty among the targeted customers increased the urgency of adopting the product development approaches. Hobbs and Shanoyan (2018) mentioned that the taste and preferences of the customers keep altering, and it is a must for the company to keep developing the product so that product utility is fetched.

Step 2: Form a Powerful Coalition

The team meeting will be organised where the financial data of Stella & Chewy's and the rival company of Tail.com will be compared. The areas of product development will be outlined to understand the urgency of adopting the alternative approaches by the business. Baloh et al. (2018) stated that developing enthusiasm among the team members are determined to be useful in planning for the change implication. Thus, Stella & Chewy's need to adopt the change, continues the discussion and then adopt the best methods for company benefit.

Step 3: Create a Vision for Change

Keeping the customer's happy and providing nutritious food to the pets is the vision that Stella & Chewy's follows. The alternation in vision by including the taste and health fusion would be helpful in attracting customers. The adding of taste to the product would not only make the pets happy but will satisfy the customers. This needs systemic training of the production team so that they can work seamlessly for the product development alternation.

Step 4: Communicate the Vision

The management will be informed about the changes using the personal meetings, and the team will be sent a notice that will include all the details written about the propped changes and the training schedules. Nikolaidis et al. (2018) stated that the mode of communication is essential in conveying a source of information. However, in the case of Stella & Chewy's the use of both verbal and written communication modes will be used to ensure that all the stakeholders are aware of the changes and its procedure to reduce the level of confusion.

Step 5: Remove Obstacle

The obstacles like the poor machinery and the waste generation need to be reduced. The execution of the sustainable measures needs to be introduced. Stella & Chewy’s needs to convince the production team to lead updated technologies so that they can contribute their best in the process of product development.

Step 6: Create Short term win

The production team and the other stakeholders will be invited for parties and get-togethers so that a flexible bond is created among them and they feel important. Tu et al. (2019) mentioned that the team bond is a must for an organisation to adopt new approaches. The coordination among the team members needs to take up the initiative so that they can build strategic measures and thus create a better bond. Cherishing the short wines will be building confidence among the team members, and they will be able to create a better prospect to win.

Step 7: Build Change

The weekly training of the production staff within the working environment will allow the team to fetch professionalism in developing innovative and tasted pet foods. The strategies measures are sure to build a positive attitude among the staff and there for a positive customer’s engagement can be developed. Jally et al. (2021) the execution of the new approaches is very tricky as the team members need to be well aware of the technologies they are using. As the profit development needs to be as per the preferences of the customers so the company will be using the proper markets evolution and the inspection of the products of the rival business to design their own exclusive pet food products.

Step 8: Anchor the change

Monitoring the performance of the business after the change implication is very much important. It has been evident that performance monitoring is essential to have a proper understanding of market performances. The sales figures will be monitor to see the changes in the sales rate with the implication of the product development approaches. The impaction of the reviews from the customer will also be monitored so that the idea about the positive success fetched by the business after the implication of the changes.

Estimated Time for The Implication of The Changes in The Operational Strategies

Figure 2: Time line

Conclusion

Stella & Chewy’s, being a successful business in the pet sector, has a huge competition. The other promising companies in the pet food industry are developing uniqueness in their product and thus lead to the creation of customer engagement. It has been evident that the business is willing to keep the customer engaged by serving them with the best product. Stella & Chewy's facing competitive changes needs to work on product development. Although product specification has been a quick, successful business idea for Stella & Chewy's but the development of the product a notch higher than that of the existing product available would be helpful in providing happiness to the pets leading to the elevation of the customer’s satisfaction. The company is sure to achieve an increment in sales rate.

Reference List

Read More

Project Management - Workshop Assignment Sample

Task Summary

You need to answer all the written questions and complete all the activities suggested. Your responses must be word processed and must be submitted to your assessor, once complete.

Provide answers to the following questions and complete all the activities:

1. Who or what are project stakeholders? Define stakeholder analysis.

2. What should be considered when identifying a project’s information requirements?

3. What communication methods could be considered for the communications management plan? Describe each of the methods, including why they would be advantageous. Summarise the inputs required to effectively manage project communications

4. What is the project management information system? What is an information retrieval system and what does it allow you to do?

5. When assessing communications management outcomes, why is the reporting performance process important? What does the process entail?

6. Think of a project example in which you can describe stakeholder interests and segment them into roles (position/ title), priority, placement, and interests to determine their forms of engagement. Also, make note of the key stakeholders. You might like to present your response in a table.

7. What is an influence diagram? Create a matrix that could show the placement of stakeholder groups and briefly explain the levels of engagement for each. List at least five general stakeholder groups/ individuals and two methods of stakeholder communication for each found in any project.

8. Read the provided case study on Shell and complete the following activities:

• Identify and list at least three Internal and External stakeholders.
• Take any two stakeholders and explain their interest. What actions can shell take to address them.
• Select any 4 stakeholders and prepare a communication plan for them.
• Complete the following stakeholder register:

9. Describe the four processes used in managing project human resources.

10. How can you align competencies with project tasks?

11. Identify and briefly outline the inputs, tools and techniques, and outputs for each of these elements of the project human resources management process.

• Human resource Planning

• Acquire project Team

• Develop Project Team

• Manage Project Team

12. Read the case study provided on Tarmac and prepare complete the following template on activity resource requirement.

Solution

Question 1

In a project, stakeholders mainly provide requirements based on the needs of the project. Identification of the key stakeholders are very crucial in a forecast. Project managers are the potential stakeholder in a project. The project manager monitors the entire project and focuses on accomplishing the project on time correctly. For Assignment Help Team members are another important stakeholder, as the quality of the project completely depends on their performance. According to Torelli et al. (2020) maintaining the resources during the conduction of the project is very crucial, as it helps to reduce the excess cost of a project. It highlights that resource management plays a crucial role and is an important project stakeholder. Investors are another stakeholder of a project, as the entire project will be carried forward based on their investment completely.

Figure 1: Stakeholder Analysis
(Source: Productplan.com, 2021)

Stakeholder analysis is a process that mainly helps to identify the stakeholders before the starting of the project. It also helps to group all of them based on their participation, involvement, and interest accordingly. This analysis helps to group the stakeholders based on four categories and that is: "High power, high interest, high power, low interest, low power, high interest and low power, low interest".

Question 2

The expectation of all the stakeholders regarding the project outcome has to be considered properly during identifying the information requirement of the project. As per the views of Yoo (2021) expert judgment and data gathering is very crucial during analysing the requirement of the forecast. After that, proper decision-making is very crucial that also helps in the accomplishment of the project correctly. The capacity and skill of each team member need to be analysed correctly and it is very crucial to make the project high graded and it is one of the major requirements accordingly.

Question 3

The communication management plan mainly considered both the formal written, formal verbal, and informal verbal communication methods. As per the guidance of Imamura et al. (2018) the formal written method is very important as it shows that all the communication will be held in the written format. It is particularly used during complex problems. The use of this method is advantageous, as it helps to plan project management and helps to communicate over long distances. Formal verbal is mainly used when there is only oral or verbal communication in between the communicators. It is advantageous as it is used during the presentation of the entire forecast. It creates more strong conversation and clear doubts related to the project completely. Another method is informal verbal. Granizo et al. (2021) stated that this type of communication is mainly used during meetings or conversions. It is done in a free environment; it highlights that all the team members easily shared their views or perspective without feeling any pressure. This method is a more flexible communication method than the formal one.

Proper communication and resource management plans are very crucial to manage project communication effectively. The stakeholder register and their engaging plan are also vital. Further, the reports based on quality, risk, and work performances contribute to managing the forecast communication correctly.

Question 4

A project management information system (PM IS) is the logical process of information that mainly contributed to execute the project effectively. Andersson (2019) stated that it is a methodical process that aids to gather information and also guides to utilize all this collected information during the conduction of the forecast successfully. PMIS aids to gather this information by using more than one software application completely.

“Information retrieval” (IR) system is a system of a set of algorithms. That played an important role and helps to show the relevance of all the important documents to searched queries. According to Rajhans (2018), this system focuses on the queries of all the users and based on that, ranks and sort all the documents correctly. This system is very important for an individual, as it provides access to individuals to journals, articles, books and also helps to protect and manage all those registers completely. It has been found that web search engines are one of the most visible applications of IR.

Question 5

Reporting performance in business is important as it enables the organisational management to identify and understand the growth potentials of the firm within the market. According to Shad et al. (2019) performance reporting is necessary within the business as it helps in the development of a strong communication network. The communication network between various stakeholders helps the managers of the firm toward the collection of proper market-based data that further help in the development of effective organizational strategies and plans. As per the statement of Kurochkina et al. (2017), performance reporting helps a firm in proper measurement of its key metrics and thereby help in mitigation of any kind of issue within the organization. Further, the stated reporting in communication management provides the performance report of the parameters of inventory, customer satisfaction, sales volume and companies growth rate in the market.

Question 6

For the project, involving the “management and use of non-renewable energy resources”, the key stakeholders of the project included the Board of Directors, suppliers, and consumers.

 

Table 1: Tabular presentation of the stakeholder’s interest
(Source: Created by the Learner)

Question 7

An influence diagram can be termed as a visual display of a decision situation. According to Zheng et al. (2017), an influence diagram consists of certain key elements that depict the decision, objectives and uncertainties through various colours and shapes. Concerning this, in a business, an influence diagram is used as a conceptualizing tool that helps to represent the causal relationship between external factors, uncertainties, decisions and the resulting outcomes.

 

Figure 2: Representation of an influence diagram
(Source: Zheng et al., 2017)

Question 8

• Stakeholder matrix


Figure 3: Stakeholder matrix
(Source: Created by the Learner)

• Method of communication

For communicating with the higher power stakeholders like the directors, mainly verbal methods of communication are followed with projects like “management and use of non-renewable energy resources”. In the verbal method of communication, scheduling of meetings, scheduling of virtual presentations and emails are followed for making communication with the “keep the satisfied group” (Brunton et al. 2017). Further, for communication with the supply chain and technical teams, again a verbal method of communication including scheduling of meetings is followed within the mentioned organisational projects. For making communication with the customers, a preferable online method using blogs, posts and campaigns are followed (Palmieri and Mazzali-Lurati, 2021). This way of communication help to generate awareness among the people on the project mission and visions. Lastly, to communicate with the legal department of the project, usually emails and fax methods are followed.

Question 9

According to the case study, employees and suppliers are the internal stakeholders and customers are the external stakeholder.

Employees and the customer are the important stakeholders of the company Shell. The performances of an employee are very crucial, as the entire presentation of the shell completely depends on the performance of each employee. On the other hand, according to Imamura et al., (2018), customers are very important for this organization. The profit rate of an organization completely depends on the sailing rate of the products and for that, raising the customer retention rate is very crucial. Similarly, fulfilling the needs and demands of each customer is very crucial, as it raises their satisfaction level and helps to create a loyal and good relationship with all of them effectively.

Communication plan:

 

Table 2: Communication plan
(Source: Created by the researcher)

Stakeholder register:

 

Table 3: Stakeholder register
(Source: Created by the researcher)

Question 10

The four major processes used in managing project human resources include:

- Plan Human Resource Management

The first process of managing human resources in a project includes proper planning. This process of managing resources involves the identification of the number of human resources and scheduling and distribution of the work among the individuals to mitigate any kind of confusion in the work (Dwivedula, 2019). This plan of action thereby helps in the proper assessment of the teamwork and also help in managing any kind of conflict.

- Acquire Project Team

The second process of managing human resources include acquiring a project team. In this process, the project members are mainly selected through advertisement jobs, interviews and hiring processes. The members for a project are selected based on their time-intensive activity and multi-decision-making criteria (Momeni and Martinsuo, 2018). This process of hiring human resources help in the development of a new team and thereby help in the proper distribution of the work to accomplish the project on the scheduled time.

- Develop Project Team

The third process of managing project human resources include the development of project teams. In this process, team members are trained with the required capabilities and constant activities to improve the performance of the project.

- Manage Project Team

The fourth process of managing project human resources include management of the project teams in which the changing role and responsibilities within a team are recorded to manage the project work. In this process, the team performance assessment is done to ensure the success of the project.

Question 11

For aligning the competencies with the project tasks, three major steps can be taken:

• Step 1: identification of the project roles
• Step 2: Assessing the project competencies
• Step 3: Assigning responsibilities to the human resources based on the competencies.

Question 12

 

Table 4: Inputs, tools and techniques and outputs for the four major processes in human resource planning
(Source: Created by the Learner)

Question 13

 

Table 5: Activity Resource Requirements
(Source: Created by the researcher)

References

 

Read More

3038IBA Business Case Assignment Sample

Overview

Imagine you are a well-known impact entrepreneur in your local region. You have been asked to prepare a short business case which presents a well-justified argument or value proposition to the senior management team of a specific enterprise or organisation outlining why and how they could involve their business practices/strategies to address a key social, economic or environmental issue that contributes to a Sustainable Development Goal.

This is NOT a financial case. The intention of the business case is to educate the decision makers in the enterprise and convince them to take action in relation to the identified social, economic, or environmental issue.

Purpose of this assessment

A key to creating business opportunities that can lead to real change is to develop well-founded and evidence-supported arguments. This assessment develops your capacity to:

• analyse the ways a specific enterprise or organisation could change and implement actions to address key social, economic, or environmental challenges to contribute to a Sustainable Development Goal

• demonstrate critical thinking and problem solving skills

• present logical, evidence-based arguments for change

• apply the knowledge presented in the course to a practical enterprise/organisation example

Solution

Introduction

Davison Canners Ltd is a renowned company of the United Kingdom which was invented by George Davison and has been providing jam, syrups, card, fruit compotes and many more like this since the year of 1995. For Assignment Help Although it has achieved great success, recently it is facing a huge problem regarding preferences in society, economy and in the environment due to acceptability and originality and sales. So, this current research is going to illustrate all these aspects.

Discussion

Issues with social challenges and its relation with the enterprise or organisation

Many people in the society are less interested in desserts and sweet foods so they are unable to be the customer of this company. On the other hand, the maximum number of people are its originality, use of organic products, and also the prices of products. So, these points are becoming a challenge to the company which is creating less profit.

Sustainable Development Goal that the challenges addressed

These challenges are creating huge problems in the field of sustainable growth. The society is focusing on social factors before buying products that are good for their health and friendly for their pockets too.The initiative of Good Health and Well Being is going to be the challenges taken up by the organisation. It has also affected responsible production and consumption (Shair et al, 2021). Besides being less effective in sales and preferences, its production is decreasing. So, these points are affecting the sustainable growth of Davison Canners Ltd.

Business opportunities that this presents to the enterprise or organisation and relevant stakeholders

More investments in their products have resulted in the invention of hot eat puddings and desserts that can attract consumers. On the other hand, the use of organic fruits and the use of the voucher of popular innovation is becoming a notable action of this company that is enhancing their business opportunity (Hlavacek, 2017). Firstly, it was started with the Irish Bramley Apples and now it is spread into one of the leading producers of fruit compotes, jam, bakery items, and many more. So, the changes with the customers' needs are a vital part that is increasing the business opportunities.

Possible action(s) the business should undertake

Firstly, the company should be more innovative in the process of delivery and it also should follow innovative ways in food production that will not affect the health of the consumers. This will ensure the social factors of the consumers is met to increase the sales. Besidesthat, Davison Canners Ltd should represent them over the internet and also should be updated in social media as per their innovations, new productions, employee and customer engagements, collecting feedback and many more like this (Tobi, Ayodele & Akindele, 2020). These few points will help to enhance their business procedure and outcomes.

Cause of being beneficial of the cations and evidence to support this

Nowadays many people are following diet charts and avoiding sweet foods. Besides that, many people are avoiding these foods due to their health issues like diabetes. So, the foods do not affect the consumers’ health and also the products are delivered safely and innovatively, the acceptance of products and services will increase. On the other hand, nowadays people are spending a lot of time on social media and the internet (Arli, 2017). So, if the updates and all delivered products are shown on the websites, then the consumers will be updated about their services. As evidence, it is identified that the sugar substitutes are 200 times sweeter than sugar but it does not give the food energy like sugar. So, the company can easily use this point for the taste issue that will not affect the health of the customers. So, if the company upgrades the packaging and delivery safety and also uses innovations that consume less time to deliver, the customers will be satisfied. Lastly, it has been shown that the internet has helped a lot to step with the customers’ requirements and preferences that has increased the product sale. This point is represented graphically below to show its significance.

Figure 1: Significance of internet in customer service
Source: (Tobi, Ayodele & Akindele, 2020)

Potential high leveled costs and benefits during the implementations of the above actions and effective evidence for it

There are many sugar substitutes and some of them are less priced than sugar and some are high priced (Allcott, Lockwood & Taubinsky, 2019). But using these substitutes, the company can bring the taste of sugar and it will not affect the consumers health and also the diabetes patient. Stevia and other artificial sweeteners contain high prices but are more useful to the customers. On the other hand, the innovation in delivery and packaging can cause high costs as there will be the use of innovative technology, extra employees and that is why companies should invest more. On the other hand, this process will help a lot to deliver safe and hygienic foods to the customer so that they become satisfied with their services and products (Nakat & Bou Mitri, 2020). Furthermore, Online advertising is a costly process that is identified as nine thousand dollars to ten thousand dollars every month and hundred thousand dollars to one hundred and twenty thousand dollars every year. Although it is the most useful and effective to spread their services and acceptance. As evidence, the cost of Stevia is 1500 per gram, which is more expensive than sugar. So, these points are a little bit expensive but they are most useful and effective for success.

Possible risks and supportive evidence

- Sugar substitutes cannot provide the energy as sugar so the customers do not get sufficient energy from this food.

- More use of sugar substitutes may cause headaches, vomiting, depression, and risk of cancer, weight gain, and many more. Medical research has explored that they are safe when they are used in moderation and a limited quantity.

- There may be a high risk of an increase of competition in online advertisements and failure will cause a huge loss. Nowadays, many new companies are delivering desserts and foods over the internet so the competition is increasing

- There are also privacy and security issues in online advertisements. As an example, it has been seen that online hackers have increased in recent days.

- Innovation in delivery and packaging may cause an extra investment that is beyond the potentiality of the company. The investment in employees numbers, their fees sometimes is beyond their capability.

Conclusion and Recommendations

The journey of Davison Canners Ltd has been very effective, good, and successful. Their products have enhanced their growth and the food quality and tastes are increasing day by day so the success is so productive. Although there are some problems which are discussed above and that is why here are some recommendations for that.

- The company should be innovative and customer-friendly. They should make the products and services innovative and should regulate with the preferences of customers.

- The company should be more careful about their employees regarding their health and well-being to promote a healthy working atmosphere and also increase productivity.

Reference List

Read More

POLITICAL DECISION-MAKING PROCESS AFFECTING GLOBAL LEADERSHIP IN INTERNATIONAL BUSINESS

Topic is Decision Making in Global Politics

Your paper should be on your content specialty area focusing on the political decision-making process affecting leadership. Content specialty areas include: (Religion/ Ministry, Math, History, English, Criminal Justice, Student Personnel Services, Information Systems, Instructional Technology, Education, Business, Nursing, Health Science).

This is a good opportunity to further research for your dissertation topic. Term Paper due November 29, 2021 at 11:59 PM EST. Paper should be ten double spaced pages, plus title page and reference page.

I Term Paper Format - APA - (Refer to the UC Dissertation Handbook Posted in Content)

Title Page

Chapter One. (Introduction)

This is where you present a short summary on the importance of your selected topic.

Chapter Two. (Review of Literature)

This where you will summarize the information and data from your references.

Chapter Three. (Procedures and Methodology)

This is where you provide your own analysis of your topic anchored in fact and reason.

Chapter Four. (Research Findings)

Summarize the meaning of your analysis and findings.

Chapter Five. (Summary, Discussion and Implications)

Practical assessment of research and implications for future study.

Reference Page

II Overall Evaluation: Excellent (A) 270 – 300; Very Good (B) 240 – 269; Acceptable (C) 210 -239; Un-Acceptable (F) &gt;210

Solution

Chapter 1: Introduction

1.1 Background

In political psychology, political decision making is one of the most renowned domains. From the post cold war period, decision making dynamics is a very important key for predicting the behaviour of different political leaders. Political pressures inside a country can have an impact on the political leaders for formulating their policies. For Assignment Help, As opined by Babic, Fichtner & Heemskerk (2017), political decision making involves complex interaction with the ideas and interests and ideologies and individuals. The current changes in government structure also impact the decision making process that consists of compelling evidence in the decision making process on different government programs. This paper focuses on how the political decision making process affects the global leader in international business.

1.2 Aim

This research paper aims to identify how the political decision making process is affecting the leadership in international business.

1.3 Objectives

- To identify the political decision making process
- To identify the relationship between global politics and international business
- To evaluate how the political decision making process impacts the leadership
- To identify the effects of political decision making in international business
- To evaluate the different strategies for analysing the political decision making process

Chapter 2: Literature Review

2.1 Political decision making process affecting leadership

Political psychology can emphasize that the leader's personality and motives and the different situations can influence the decision-making process. Many political psychology scholars also stated that political leaders use the elite to decide which group processes and media are important. It has been identified that complex political decisions sometimes influence leadership personality factors and propaganda. As Berry & Fowler (2021) coined, political psychology also contributes to understanding complex decision-making by introducing rational theories. Whenever a political leader makes decisions, stress provides an impact that entails complex decision-making; however, this curvilinear change affects the decision quality. Several studies have been made on different political personalities and leadership styles. It has been identified that leaders' personal history also contributes to their personality and leadership style; however, personality traits also affect leaders' policy preferences.

Political leaders are mainly responsible for establishing a process through which all the decisions will be made. As suggested by Johnson & Orr (2020), this process is severe for the country or world as it can hinder and facilitate job execution and increase the overall effectiveness of the decision. Before making any decision, it is very important to evaluate the general nature of any leader's culture and decision-making style, which can help make the decision-making process more effective. Leaders can also recommend strategies that should be involved in the overall decision-making process, considering how it will affect the rest of the world.

2.2 Effects of political decision making in international business

The political decision-making inside a country or world generates laws and legislation for controlling the business environment. All the companies doing business inside a specific country or internationally must follow or comply with this regulation, or they can face hefty penalties. All these laws and regulatory requirements indeed reflect the social value and objectives of any specific country. However, regional differences can also impact the business. All the global companies doing business in different continents need to obey these laws. For different countries, laws are different, so they need to adjust their business structure and procedure according to the host country (Howarth & James, 2020). The government leader mainly implements these policies and legislations, and the political stability of any country also directs the business activity.

For example, Apple is a global tech giant, and recently they stopped charging their phones. Asian and American countries do not have any legislation or rules regarding it; however, in European countries, they have strict laws and quoted the company that if they need to do the business inside the region, they have to provide chargers for their customers. This is how the political decision impacts international business. Apart from these things, sometimes political stability is important for conducting business activities. As suggested by van der Zwan & Nijhuis (2019), if any uncertainty or political disruption takes place, then it will be difficult for the business to conduct their day to day activities as they have to face problems to perform their routine activities such as purchasing equipment or getting investment from the stakeholders.

2.3 Theories

2.3.1 Decision Theory

Decision theory is related to underlying choices such as agent choices. This is also a mundane choice when a person thinks about whether it is important to take a bus or taxi. The decision theory is mainly structured with normative decision theory, which evaluates the preferences and attitudes that must be justified by any leaders personality and generic circumstances. As said by Davis Sramek, Thomas and Fugate (2018), this theory also sets accounts of rationality and has more substantial questions about the leaders desire and beliefs on an ongoing situation. However, this theory also demonstrates an uncertainty risk present on which the value can be understood according to the leaders decision. Preferences and prospects are the main Central concept of this decision theory, which clarifies that the preference has a comparative attitude. Decision theory also stated how different leaders make decisions based on risk and uncertainty. It analyses and sets axioms that define how different individuals behave when they face any challenges in the empirical and theoretical ground. However, political decision making forced us to adapt and apply the different rational choices for the private or governmental organisation efficiently.

Chapter 3: Methodology

3.1 Research Design

The main aim of the research is to evaluate the impact of the political decision-making process in global leadership in business. Here, in the research for gathering accurate and transparent data on research, a descriptive research design has been considered. The characteristics of descriptive research design for non qualified issues helped the researchers analyse the problems faced while performing the global leadership approaches in business. The descriptive research design also can help in completing the research in a more time efficient way. Here, for finishing the research more quickly, a descriptive research design has been considered. It also helps in observing all the phenomena of research in a more detailed way. For collecting the data more naturally, a descriptive research design has been considered.

3.2 Data collection Method

The research paid attention to how the political decision-making process affects the global leader in international business. For the process of gathering data on the research, secondary qualitative data collection procedures have been considered. Voleti (2019) opined that secondary data could be collected from both internal and external sources. For cost-effective and time efficient characteristics of secondary research here, secondary research methodology has been selected that helped the research complete within a limited time and a limited cost. Secondary research methodology has been conducted to incorporate a wide range of information about the research topic. Choosing secondary data on research also helped in working with a set of pre-existing data on the research. The data for the research has been collected from several journal articles, websites that are related to the topic.

3.3 Data Analysis Method

For the data analysis to identify the impact of political decision making procedures on the leadership approaches in global business, thematic analysis has been chosen. As coined by Nowell et al. (2017), thematic analysis can be recognised as one of the most appropriate approaches for qualitative data analysis. Thematic analysis helps in producing flexibility in research that can help in incorporating more analytical options. Here, for giving the research more detailed analytical options, thematic analysis has been considered. The quick and easy characteristics of the method helped in analysing the data with a quicker process. It also helps in exploring the qualitative data on research. For more exploration of the qualitative data on the topic, a thematic analysis has been considered.

3.4 Research Ethics

Reducing the legal and contractual consequences on research to maintain the privacy of the research became a prime concern. In the process of collecting secondary data, only authentic websites have been considered. As per the views of Dooly et al. (2017), before starting a research, it is important to ensure that the research will not harm anyone. No patent has been used. For gathering information, no transaction has been preferred. Only free websites have been preferred to collect secondary data on the research. Information has been collected while maintaining The Privacy Act 1988 (Australian government, 2019). After finishing the research process, all the data on the research has been deleted. No animals were being hurt during the data collection procedures.

Chapter 4: Analysis & Findings

Theme 1: Areas covered by the political decision-making process

Political decision-making can be considered as one of the main domains in political psychology. Several theoretical frameworks have been used for creating the decision-making process. It includes the process of making decisions by parties and the government. The political decision-making procedures pay attention to the social aspects. As opined by Christensen et al. (2020), process satisfaction, the index of legitimacy, the confidence in political decision-making, and support can be considered as the variables for capturing the legitimacy in the decision making procedures. It also helped in providing advice on the effect of several events that created several social problems. The political decision-making procedures help in making initiatives that can help in reducing the impact from the society. The decision making process also helps in reducing a certain impact on the health care systems. It helps in making the decisions for diminishing the impact on health care systems. It also can help in making decisions to reduce and to manage the economic fallout in the society. It also examines the impact of a certain situation in the business, employment, loans at the international level, micro and macro economic development, and others. Decision making helps ensure each of the responsibilities with the help of parliament, political stakeholders, and others.

Theme 2: Roles and responsibilities of persons associated with the political decision-making process

Choosing the right people in the decision making procedures can be considered one of the main approaches to success in the decision-making procedures. There is also the necessity of understanding the effectiveness of the decisions and understanding the roles and responsibilities that can help produce effective decisions. There are certain roles among the people who have participated in the decision making procedures. As stated by Kaszkur & Kapsa (2019), citizens are also included in the decision-making procedures. They are the chair, ministers, senior party leaders, the secretary, subject matter expert, senior information officer and communication advisor, external stakeholders, Governmental representative, financial expert, and legal expert. The chair of the decision making body has the responsibility of leading and coordinating. The chair also plays a crucial role in communicating with the citizens and the external stakeholders. Ministers play an important role in providing sector specific expertise. The secretary has the responsibility of developing the rules for better decision making procedures. The senior information officer helps gather the information from the different sources that will help produce effective decisions. A communication advisor helps in communicating political decisions with citizens, media, and also stakeholders. The political leader completes the responsibility of managing the political process. The external stakeholders also play a crucial role in the process of making political decisions. The security advisor helps in providing the solutions for the security-related issues in the decision-making procedures. The financial expert pays attention to the financial activities needed to be performed in the decision making procedures. On the other hand, the legal experts include legal advice on the decision making procedures.

Chapter 5: Summary, Discussion, and Implication

5.1 Discussion

5.1.1 Relation between global politics and international business

The world has entered the era of global order changes, and the population and technology are increasing rapidly and maintaining a strong connection between the people. On the other hand, the government faces several issues and threats such as climate change, pollution, and covid19. Suedfeld & Tetlock (2019) stated that geopolitics is one of the most important topics, which implies international laws that help the government maintain different businesses inside their territories. There is a close relationship between global politics and international business, which helps both government and business to maintain a healthy business environment and allows the government to control different international businesses from operating inside the country. Several businesses are doing infrastructure-related programs, which allow the country e to get a financial boost. Geo political interest for the companies makes a difference in the international business environment and employees’ legal requirements for eliminating potential threats of war or terrorism. As Niessen (2019) coined, it also helps to make a momentum inside a country to take the initiative and mechanism for engaging other countries in different parts of the world. Sometimes the business faces challenges, and they want help from the government, and under the policies and legislation, the government provides the business with the help they require (Sniazhko, 2019). This is the reason why the businesses can run smoothly inside different territories.

5.1.2 Changes in international business due to decision making global politics

Global politics has a huge impact on international business and trends by changing the business policies inside any specific country or region. It has been identified that environmental changes and unemployment, nationalistic pressure, and poverty are some major reasons why the government changes its policies. Pugliese & Senna (2018) opined that sometimes it brings more restrictions for any specific type of business, and foreign companies or domestic companies need to change their business process to cope with the changes. Companies indeed need to change their business process; otherwise, they will not get finance and enough business from that specific market. On a similar note, sometimes policy changes can have a favourable impact on the overall business environment. To run the global business and trade smoothly, different governments are working together, and sometimes trade acts as a political tool that can cause a trade war or embargo. Pritzlaff Scheele & Nullmeier (2018) stated that this is one of the main reasons why international businesses sometimes face several changes in business regulations that add additional costs for the big international companies. Foreign example, due to the increase in pollution, several governments put restrictions and imposed a ban on coal power electricity plants. It is an effective change; however, it damages the business of power plants. On the other hand, the government lifts off restrictions and gives additional information to car manufacturers who produce electric cars. So, this decision making is favourable for the electric car business, and this is how global politics is impacting international business.

5.3 Summary

The research mainly aims to identify the effect of political decision-making processes on the leadership styles and approaches of international business entities. The research can be considered an appropriate approach for evaluating the political decision-making process that has a greater impact on the leadership of international business activities. The provision of a clear background helped the leader in getting a broader assumption about the topic. From the analysis of the literary sources, it can be found that the political decision-making process plays a crucial role in determining the leadership styles in global business organizations. The effects of the political decision making process on the operational activities of international business. The inclusion of several methodological approaches also helped in making the research more appropriate.

5.4 Practical assessment of research and implication for future study

The research can be considered an appropriate approach for identifying the impact of the political decision-making process on international business leadership. The unexplored areas and the flaws of the research helped in identifying the future scope of the research. The research only considered secondary method research methodology. The inclusion of primary research methodology added the future scope of the detailed analysis of the topic. The limitation of time and cost also affected the projection of the research in case of conducting primary research it will help in operating realistic data on the research.

Conclusion

The main aim of the research is in evaluating the impact of the decision making in political aspects and the impact of the decisions in the global leadership of the international business. Here for getting accurate data on the research process, secondary data collection methodology has been considered. The information has been collected from different secondary sources. The inclusion of the analysis of the literary sources can significantly impact the leadership approaches in global business. The effect of the global political decisions on the leadership approaches of the business has been included. Thus, it can be concluded that appropriate political decision making procedures are an effective approach for the growth of the business also.

References

Read More

Corporate Governance of Hyper-Energy Ltd Assignment Sample

Instructions

1 Your assignment should address the question(s) and stated learning outcomes by drawing on appropriate learning from Modules 1 to 6 of the subject materials.

2 The assignment is an opportunity to demonstrate your learning, including theory and its practical application. To address the question(s) in this assignment, you are to draw from the core materials in the subject notes, prescribed readings and textbooks, and the resources provided in the Governance Institute Learning Management System but should not be limited by these materials.

3 The assignment counts for 30 per cent of your total mark in this subject.

4 The total word limit for this assignment is 3,000 words. You are cautioned that there are penalties for exceeding the word limit, namely a deduction of marks. There will be no penalty if the assignment exceeds this word limit by 10 per cent or less.

However, for every 500 words or part thereof beyond the stipulated assignment word limit of 3,000 words, ten marks will be deducted from the total of 100 marks available for the assignment. You must declare the total number of words (excluding footnote references) in your assignment.

5 The Assignment Assessment Rubric provides guidance on the criteria and performance indicators against which your submission will be assessed.

6 Answers must contain original thought and ideas. You must acknowledge, through accurate in-text citation and referencing, any idea supported by or contrasted with the work of another author. Note that academic work created by the use of Artificial Intelligence (AI) online software or a contract cheating website service (a website posing as a ‘study support’ or similar service) and claiming it as your own work, will be subjected to an allegation of academic misconduct investigation per the Student Academic Misconduct Policy.

7 The date for submission is Tuesday, 2 May 2023.

Objectives

This assignment is designed to:

- Review your knowledge of the relevant principles of corporate governance as found in the ASX Corporate Governance Council Corporate Governance Principles and Recommendations, 4th edition (Principles and Recommendations) and test your understanding of these principles and how they might be applied to a particular organisation.

- Assess your capacity to identify and analyse the elements of good corporate governance for companies that operate in particular circumstances and are planning to undergo change. In particular, you should identify and apply the following aspects:

− the role of the board and its relationship with management, including how the board can be as effective as possible during a time of change

− risk management issues arising in the particular industry

− the role of the board in ensuring timely disclosure to the market

− the membership and structure of the board, including the composition of the board and the role of any committees that should be established, and

− the role of the board to produce a system of governance which will assist the company to successfully navigate through a period of change.

Solution

Question 1:

1.1 The important functions of the board of Hyper

The board of Hyper performs crucial roles amidst a period of notable transformation, which entails supervising and directing the enterprise's strategic trajectory, mitigating risks, and guaranteeing that the company conducts its operations in a conscientious and principled manner. For Assignment Help, in the specific context of the company's plan to augment its retail operations by incorporating compact 'supermarket-type' stores into its current distribution centres, the board ought to take into account the subsequent factors:

Strategic oversight: The governing body of Hyper assumes a pivotal function in establishing the enterprise's strategic trajectory and overseeing its execution. When contemplating the expansion of its retail stores, the company's management team ought to consider various factors, including but not limited to the magnitude of the market opportunities, the competitive environment, and the company's current capabilities and resources (Nili, 2022, p.92). It is recommended that the board oversee the evaluation of advancements made towards the strategic objectives and goals and make modifications to the strategy as deemed appropriate in light of evolving circumstances (Nguyen, 2021, p.78).

Risk management: As Hyper seeks to broaden its business activities, the board must exercise caution in recognising and regulating the hazards linked to the expansion. The potential risks involved in a business operation may encompass operational risks, financial risks, and reputational risks. Operational risks may arise from supply chain disruptions, while financial risks may stem from price hikes or pricing pressures (Wang et al. 2020, p.36). Reputational risks may manifest in the form of adverse publicity or dissatisfaction among consumers. It is imperative that the board guarantees the implementation of suitable risk management procedures and mechanisms and that these are periodically assessed and revised as required (Nili, 2022, p.79).

Corporate responsibility: Hyper bears the obligation to conduct its operations in an environmentally and socially conscious fashion while considering the concerns of all stakeholders (Dhar et al. 2022). It is recommended that the board of directors take measures to make sure that the company has established suitable policies and procedures to regulate its conduct and that these are efficiently disseminated and executed across all levels of the organisation. This could encompass domains such as ecological sustainability, corporate social responsibility, and moral business conduct (Beji et al. 2021, p.150).

The decision regarding if the board should modify its role to align with the evolving business landscape is contingent upon the unique circumstances of the organisation and the characteristics of the transformations. Nevertheless, certain overarching principles and guidelines may prove beneficial in facilitating this determination.

The ASX Corporate Governance Fundamentals and suggestions can offer a valuable framework for implementing appropriate corporate governance measures. According to Principle Two of the ASX Principles, the composition and responsibilities of the board should be such that it comprises a suitable blend of abilities, expertise, and diversity, thereby facilitating the effective discharge of its duties (Czernkowski et al. 2019, p.720).

In general, it is recommended that the board of Hyper adopt a proactive and strategic stance towards overseeing the company's expansion efforts while simultaneously upholding rigorous standards of corporate governance and adhering to ethical and responsible business practices. Through this approach, it can facilitate the attainment of enduring prosperity and viability of the enterprise, concurrently engendering worth for every concerned party (Cooke et al. 2019, p.78).

Moreover, regarding the potential modification of the board's role to align with the evolving nature of its business, those crucial factors need consideration, and these are discussed widely in the following:

Board compositions: As previously stated, it is imperative that the board of Hyper possesses a suitable combination of competencies, expertise, and inclusivity to facilitate proficient supervision of the organisation's broadened retail endeavours. The implementation of this strategy may necessitate the enlistment of novel directors possessing pertinent proficiency or the formation of consultative committees to furnish direction on particular matters (Merendino & Melville, 2019, p. 75). Given the growing popularity of its retail business, it may be necessary for the board to consider the recruitment of new directors who possess specialised knowledge in retail-related areas, including but not limited to retail design, retailing, and customer experience.

This would aid the board in gaining a more comprehensive comprehension of the challenges and prospects associated with the new enterprise, thereby enabling them to deliver efficient supervision (Fernández-Temprano & Tejerina-Gaite, 2020, p.67). As Hyper allocates resources towards the establishment of new stores and broadens its operational scope, the board will be required to evaluate the financial ramifications of these strategic choices. The task at hand may necessitate the involvement of directors possessing specialised knowledge in the areas of finance, accounting, and handling risks (Merendino & Melville, 2019, p.78). It is imperative for the board to endeavour towards achieving diversity in its composition to facilitate the consideration of a broad spectrum of experiences and points of view. Those mentioned above may encompass diversity with regard to race, ethnic background, age, and professional experience (Fernández-Temprano & Tejerina-Gaite, 2020, p.65).

Board procedures: It is possible that the board will have to make modifications to its procedures and processes to accommodate the alterations in the organisation's operations. As an illustration, it could be imperative to convene meetings more frequently or obtain more comprehensive progress reports from the management regarding the expansion. It is recommended that the board consider implementing novel reporting procedures to guarantee the prompt and pertinent dissemination of information regarding the efficacy of the recently established retail operations (Kao et al. 2019, p.190).

Those, as mentioned earlier, may encompass periodic assessments pertaining to the performance of the store, prevailing sales patterns, and feedback from customers. It is recommended that the board convene more regularly to ensure effective monitoring of the company's advancement and to deliberate on matters pertaining to its expansion (Baker et al. 2020, p.236). As an alternative course of action, the board may institute a subcommittee that would be responsible for supervising the retail operations.

At the same time, this subcommittee would convene independently from the primary board. As previously mentioned, the board must exercise caution in recognising and controlling potential hazards linked to the expansion. In order to guarantee efficient risk management, it may be necessary for the board to institute novel risk management procedures and safeguards and continuously assess the efficacy of said procedures (Kao et al. 2019, p.191).

Ultimately, the board must engage in routine evaluations of its own performance in order to ascertain whether it is furnishing proficient supervision and direction to the organisation. The process may encompass self-evaluations, evaluations by peers, or evaluations conducted by a specialist in corporate governance. The outcomes of these assessments ought to guide any modifications to the board's structure, procedures, or ethos (Baker et al. 2020, p.240).

Board cultures: The board should also cultivate an environment that promotes transparent communication, wherein board members are motivated to inquire, contest presumptions, and express their viewpoints. This measure will facilitate the board's ability to make well-informed decisions regarding expanding retail operations while promoting inclusivity and mutual respect among all board members (Newman & Ford, 2021, p67).

The governing body ought to assume responsibility for its determinations and conduct. This entails guaranteeing that directors possess comprehensive knowledge of the firm's performance and hazards and undertake suitable measures to tackle any emerging challenges. In addition, it encompasses the responsibility of being responsible to investors and other stakeholders and ensuring transparency in the board's decision-making procedures (Bilan et al. 2020, p.70).
The board must put an emphasis on doing the right thing and making choices that are consistent with the company's principles. That means running an honest and open business in accordance with all applicable rules and regulations.

Finally, the board should prioritise improving itself and the firm. If the board is to manage the business's operations and strategy properly, it must frequently assess its own makeup, methods, and culture, making adjustments as necessary (Newman & Ford, 2021, p.198). The process also involves polling shareholders, customers, and workers for their input to determine where the firm is falling short and where it is succeeding.

In conclusion, it can be said that Hyper's board of directors plays a crucial part in the company's overall strategy for growing its retail presence. In order to ensure that Hyper continues to grow in a sustainable and ethical way, the board of directors should adopt an anticipatory and proactive strategy for managing the company's development (Bilan et al. 2020, p.243).

Question 2

2.1 Determination of Hyper’s appropriate mix of skills and diversity:

To determine whether Hyper’s board has the perfect mix of diversity and skills, the following steps can be useful:

Conduct skills audit: The audit of skills is a systematic evaluation of the competencies, knowledge, and proficiencies of every board of directors member. The implementation of a skills matrix can facilitate the identification of both the existing proficiencies and expertise within the board, as well as any potential gaps in these areas (Duchek et al. 2020, p. 389). The skills audit should take into account the proficiencies and knowledge necessary to facilitate the organisation's strategic trajectory, encompassing the foray into the retail sector. The board may consider the proficiency and knowledge of the recently designated managers with prior experience in the retail industry to discern any deficiencies that require attention (Amyar et al. 2019, p. 389).

Assess diversity: The evaluation of diversity entails an examination of the board's makeup with respect to various diversity dimensions, such as race, age, gender, and other related factors. A diversity matrix can be utilised by the board to evaluate the extent of diversity and pinpoint any potential areas that require enhancement (Duchek et al. 2020, p. 390). It is recommended that the board of directors take measures to ensure that it accurately represents the variety of the stakeholder group, including consumers, staff members, and shareholders. This measure will aid in guaranteeing that the decisions made by the board are comprehensive and considerate of the concerns and viewpoints of all parties involved (Gomez & Bernet, 2019, p.390).

Consider the impact of retail growth: The firm’s board should consider the potential effects of retail development on the composition of the board of the firm in relation to diversity and skill. The maximisation can require the board of Hyper’s directors who have skills in retailing, activities of promotions, supply chain systems, and logistics. The firm’s board can also deliberate on the possible influence of the development on the risks of the business and the importance for the directors to have knowledge of risk management (Fountaine et al. 2019, p. 89).

Review board policies: To ensure that the board's policies are consistent with the values of diversity and inclusion, a review should be conducted. The board may want to examine its hiring practices to make sure they are welcoming to all kinds of applicants. To ensure that directors are afforded enough chances for learning and development that foster diversity and inclusion, the board may choose to reevaluate its policy on education and training (Fine et al. 2020, p.70).

Involvement with stakeholders: The process of engaging stakeholders comprises asking for feedback from those with a vested interest in determining the best composition of board members in terms of skills and diversity. The board may consult with employees, customers, and stockholders via surveys, focus groups, and other means of communication (Song et al. 2020, p.1029). Incorporating the viewpoints and perspectives of stakeholders is crucial to inform the board's decision-making process and to ensure that the goals and principles of stakeholders are represented.

At the same time, the board should seek feedback from various stakeholders, including customers, shareholders, and employees, to gain insight into their viewpoints regarding the composition and structure of the board. Conducting an assessment of the board's performance can facilitate the identification of areas that require improvement (Pucheta Martinez and Gallego Alvarez, 2019, p. 89). Additionally, such an assessment can enhance the board's reliability and credibility with participants. The board may consider implementing various methods, such as conducting surveys, organising focus groups, or utilising social media or other communication channels to engage with stakeholders (Song et al. 2020, p.1022).

Regularly evaluate the board performance: The board must engage in consistent evaluations of its own performance in order to ascertain whether it is furnishing proficient supervision and direction to the organisation. The process may entail various methods such as self evaluations, peer assessments, or external evaluations conducted by a specialist in corporate governance. The outcomes of these assessments ought to guide any modifications to the board's structure, procedures, or principles (Gomez & Bernet, 2019, p.27).

Overall, it can be said that by adhering to the prescribed procedures, the governing body of Hyper can guarantee that it possesses a suitable blend of competencies and inclusivity to bolster the organisation's strategic pattern, at the same time it can ensure that the board of Hyper resolutions are comprehensive and considerate of the concerns and viewpoints of all parties involved.

2.2 Steps to deal with challenges:

To ensure that Hyper’s composition and structure allow it to deal appropriately with the challenges it faces, the firm must take the following eight steps:

Review the strategic direction and challenges: It is recommended that the board undertake a comprehensive evaluation of the organisation's strategic orientation and ascertain its obstacles. This will help the board in finding the essential experience and skills required to manage such issues effectively. For example, if a company tends to broaden its activities into the markets, this can be important for the board to appoint new directors who have the expertise in global business and marketing (Benbya et al. 2020, p.37).

Evaluate the board’s structure and composition: The board of Hyper also must evaluate its composition and structure to discover its appropriateness for achieving its organisational objectives. The evaluation should consider the competencies, professional background, and proficiency of every director and assess whether these are congruent with the organisation's strategic orientation and predicaments. Suppose a company is seeking to expand its operations into retailing. In that case, it may be necessary for the board of directors to seek out individuals who possess a background and skill set in retailing or marketing (Wessel et al. 2021, p.128).

Find gaps in experience and skills: After assessing the composition and structure of the board, it is recommended that the board identifies any potential deficiencies in abilities or previous experience. This approach will assist the board in identifying the requisite skills and experience necessary to address the gaps, as mentioned earlier. In the event that a company intends to expand into a new market, it may be necessary for the board to seek out directors who possess relevant experience in the declared market (Grimes et al. 2019, p.840).

Review the appointment procedure: The process of appointing board members is crucial in guaranteeing that the board of directors comprises individuals possessing the necessary expertise and knowledge to tackle the obstacles confronting the organisation effectively. It is recommended that the board undertake a review of its selection process to ascertain its suitability and ability to attract a varied pool of candidates. Job descriptions, hiring practices, and selection criteria all need to be assessed for accuracy (Kulkarni & Anantharama, 2020, p.56).

It is advised that the job title be reviewed carefully to ensure that it accurately represents the qualifications for the position. Reviewing the hiring procedure may help determine whether or not it is successful in drawing applicants from all backgrounds, especially those from underrepresented groups (Benbya et al., 2020, p.204). It is suggested that the criteria for selection be carefully examined to ensure that they are objective, transparent, and well-defined.

Consider the firm’s risk profile: The firm’s board of members should check that it has enough resources to cope with any risks. Involving a risk panel or knowledgeable board members might be helpful here. Depending on the nature of the company's risks, the types of knowledge and experience that should be represented on the board will change. For instance, in a highly regulated business, the board may benefit from having directors with experience in safety and regulatory affairs (Wessel et al. 2021, p.110). A similar requirement for directors skilled in financial risk management may arise if the firm operates in a particularly unstable market.

Review the board committee: To make sure the readiness for the business's strategic direction and difficulties, the board should examine its committee structure. In certain cases, this may call for the dissolution of existing committees and the formation of new ones.

The strategic focus and problems facing the business should inform the board's committee set-up. For instance, if the firm is venturing into retail, the board may have to form a separate committee to manage the retail division (Grimes et al. 2019, p. 192). Similarly, the board can have to form an oversight committee to manage regulatory matters if the firm operates in a highly regulated sector.

Implement a program for director growth: To guarantee that board members get the education and experience they need to do their jobs well, the board should institute a director development programme. Governance, handling risks, and other related courses may be included.

It is possible to increase confidence in directors' abilities and performance by investing in a director development programme. The programme should cater to the board's unique requirements and incorporate formal and casual learning settings (Kulkarni & Anantharama, 2020, p.89). Directors may improve their abilities and expertise by, for instance, participating in conferences, seminars, and workshops.

Regular review and refresh of the board: The board's composition should be evaluated and updated on a regular basis to ensure it continues to serve its function effectively. This may call for the resignation of current board members or the selection of new ones.

The board's continued relevance to the strategic direction and issues facing the organisation may be ensured by periodic reviews and renewals of its membership. Assessing the knowledge, abilities, and experience needed to meet the issues facing the organisation should be part of an open and impartial evaluation process (Benbya et al. 2020, p.89). This might entail the resignation of current board members or the appointment of fresh candidates to fill vacancies.

In conclusion, Hyper will be able to fill its board with directors who have the knowledge, experience, and competence to help the firm overcome the obstacles it confronts. The board will be better able to make choices in line with the business's overall strategy and strengthen the company's position to achieve its goals. Let's go through each stage in further depth.

References

Read More

Sample Category

Assignment Services